ATLS10 2019/2020 Post-Test ATLS10 exam questions with answers (answers outlined!!) 1. A 24-year-old male pedestrian, struck by an automobile, is admitted to the emergency department 1 hour after injury. His blood pressure is 80/60 mmHg, heart rate 140 beats per minute and respiratory rate is 36 per minute. He is lethargic. Oxygen is delivered via face mask, and two large-caliber IVs are initiated. Arterial blood gases are obtained. His PaO2 is 118 mmHg (15.7 kPa), PaCO2 is 30 mmHg (4.0 kPa), and pH is7.21. The treatment of his acid-base disorder is best accomplished by: a. Hyperventilation b. Restoration of normal perfusion c. Initiation of low-dose dopamine d. Administration of sodium bicarbonate e. Initiation of phenylepinephrine infusion 2. The highest priority in managing a patient whose injuries include closed extremity fractures is: a. Assesing limb perfusion b. Preventing necrosis of the skin c. Decompressing compartment syndrome d. Addressing respiratory insufficiency Identifying crush syndrome 3. A 34-year-old female is involved in a motor vehicle crash is brought to the emergency department. She is talking, but her voice is hoarse and on exposure she has diagonal bruising of the chest and anterior neck. What is the next step?
a. Direct laryngoscopy to exclude laryngeal trauma b. Oxygen by non-rebreathing mask c. Protecting the spine by making her lie down d. Palpation of the anterior neck e. Attaching a pulse oximeter to her finger 4. A 30-year-old male sustains a gunshot wound to the right lower chest, midway between the nipple and the costal margin. He is brought by ambulance to a hospital that has full surgical capabilities. In the emergency department he is endotracheally intubated, fluid resuscitation is initiated through two large-caliber IV lines, and a
closed tube thoracostomy is performed, with the return of 200 ml of blood. A chest xray reveals correct placement of the chest tube and a small residual hemothorax. His blodd pressure is now 70/0 mmHg, and his heart rate is 140 beats per minute. His hypotension is most likely due to: a. Tension pneumothorax b. Massive hemothorax c. Pericardial tamponade d. Intraabdominal bleeding e. Insufficient isotonic crystalloid infusion 5. A 20-year-old athlete is involved in a motorcycle crash after having ridden for hours on a very hot day. When he arrives in the emergency department, he shouts that he cannot move his legs. On physical examination, there are no abnormalities of the chest, abdomen, or pelvis. The patient has no sensation in his legs and cannot move them, but his arms are moving. The patient’s respiratory rate is 22, heart rate is 88, and blood pressure is 80/60 mmHg. He is pale and sweaty. What is the most likely cause of his hypotension? a. Neurogenic shock b. Cardiac tamponade c. Myocardial contusion d. Hyperthermia e. Hemorrhagic shock 6. Comapred with adults, children have: a. A longer, wider, funnel-shaped airway b. A less pliable, calcified skeleton c. Greater mobility of mediastinal structures d. A relatively smaller head and larger jaw e. Anterior displacement of C5 on C6
7.
Which one of the findings below requires a definitive airway in trauma patients? a. Facial lacerations b. Repeated vomiting c. Partial thickness facial burns, cough, and hoarseness d. Sternal fracture e. Glascow Coma Scale score of 12
8. In a patient with spinal cord injury, sacral sparing: a. Refers to a fracure of the sacrum b. Is part of the spinal shock syndrome c. Is a good prognostic sign d. Indicates a complete spinal cord injury e. Occurs only with complete transection of the lumbosacral spinal cord
9. A 22-year-old woman falls whil skiing. She is evaluated at a small community hospital that does not have neurological services. Spinal motion is restricted, supplemental oxygen by mask is administered, and two antecubital IVs are placed. Her Glascow Coma Scale score is 12, pupils are equal, blood pressure is 135/76 mmHg, heart rate is 105, and respiratory rate is 19. Chest x-ray is normal. This patient’s management priorities are: a. Repeat primary survey and transfer to a trauma center b. Definitive airway, CT of the head, and intracranial pressure monitor c. IV mannitol, definitive airway, CT of the head, and neurosurgery consult d. CT of the head, EEG, cerebral perfusion pressure monitoring, and hypertonic saline e. IV Dilantin, IV mannitol, mild hyperventilation, and serial arterial blood gases 10. A young male patient is brought to the emergency department following a 5-meter (16-foot) fall from a roof. He responds to pressure by pushing away your hand, opening his eyes, and verbalizing inappropriate words. Pupils are equal. The most important step is management of this patient would be:
a. Immidiate intubation to protect his airway b. Administer 25 mg/kg IV bolus mannitol c. Insert two large-bore IVs d. Alcohol and drug screening e. Determine whether amnesia is present and, if so, for what period of time 11. Twenty-seven patients are seriously injured in an airplane crash at a local airport. The principles of triage include: a. Establish a triage site whthin the hot zone of the crash site b. Treat only the most severely injured patients first c. Immidiately transport all patients to the nearest hospital
d. Treat the greatest number of patients in the shortest period of time e. Produce the greatest number of survivors based on available resources 12. A 35-year-old female falls down a flight of stairs. She has extensive bruising of her face and head. Her heart rate is 120, blood pressure 90/70 mmHg, and respiratory rate is 26. The patient`s condition is most readily explained by: a. Associated head injury b. Hypovolemia from hemorrhagic shock c. Alcohol intoxication d. Spinal shock from cervical spine injury e. Neurogenic shock from cervical spine injury 13. Which one of the following statements is correct: a. Cerebral contusions may coalesce to form an intracerebral hematoma b. Epidural hematomas are usually seen in the frontal region c. Subdural hematomas are caused by injury to the middle meningeal artery d. Subdural hematomas typically have a lenticular shape on CT scan e. The associated brain damage is more severe in epidural hematomas 14. An 18-year-old is brought to the emergency department after having been shot. He has one bullet wound just below the right clavicle and another just below the costal margin in the right posterior axillary line. His blood pressure is 110/60 mmHg, heart rate is 90 beats per minute, and respiratory rate is 34 breaths per minute. After ensuring a patent airway and inserting two large-caliber IV lines, the next appropriate step is to: a. Obtain a portable chest x-ray b. Administer a bolus of additional IV fluid c. Perform a laparotomy d. Obtain an abdominal CT scan e. Perform diagnostic peritoneal lavage
15. Which one of the following statements is true concerning cranial anatomy related to traumatic injury? a. A scalp laceration is an unlikely source of bleeding in a patient with a long transport time. b. The meninges are comprised of the dure, pia, and arachnoid. c. The middle meningeal artery lies between the dura and pia mater. d. Effacement or shift of the ventricles occurs commonly with concussions.
e. A dilated pupil is produced when the sympathetic nerve fibers located on the third cranial nerve are compressed by the temporal lobe. 16. A 47-year-old house painter is brought to the hospital after falling 6 meters (20 feet) from a ladder and landing straddling a fence. Examination of his perineum reveals extensive ecchymosis. There is blood in the external urethral meatus. The initial diagnostic study for evaluation of the urinary tract in this patent should be: a. Cystoscopy b. Cystography c. Intravenous pyelography d. CT scan e. Retrograde urethrography
17. Which of the following statements is true? a. The laryngeal mask airway is an infraglottic device. b. The multilumen esophageal airway occludes the supraglottic lumen and ventilates through the port placed distal to the vocal cords. c. The nasopharyngeal airway is an ideal supraglottic device for patients with cribiform plate fractures. d. Nasotracheal tubes position a cuffed airway in the infraglottic space. e. Tracheostomy tubes are placed in apneic, hypoxic patients in the supraglottic space.
18. Neurogenic shock has all of the following characteristics except which? a. Hypotension b. Vasodilation c. Bradycardia d. Neurologic deficit e. Narrowed pulse pressure
19. Which one of the following statements concerning spinal cord trauma is true? a. A normal lateral c-spine film excludes injury. b. A vertebral fracture or dislocation is unlikely in the absence of physical findings of spinal cord injury. c. A patient should remain on a long board until injury is excluded radiographycally. d. Diaphragmatic breathing in an unconscious patient whi has fallen may
indicate cervical spinal cord injury. e. Determination of wether a spinal cord lesion is complete or incomplete must be made in the primary survey. 20. Which one of the following statements is false concerning Rh isoimmunization in pregnant trauma patient? a. It occurs in blunt penetrating abdominal trauma b. It is produced by minor degree of fetomaternal hemorrhage c. Rh immuniglobulin therapy should be administered to pregnant females who have sustained a gunshot wound to the leg d. Isoimmunization is not a problem in Rh-positive pregnant patients e. Initiation of Rh immunoglobulin therapy does not require proof of fetomaternal hemorrhage
21. An 18-year-old motorcyclist collides head-on with a pick-up truck and sustains massive facial injuries. In the emergency department his blood pressure is 150/88 mmHg, heart rate is 88 beats per minute and regular, and respiratory rate is 26 breaths per minute. His respirations are labored and sonorous. His Glascow Coma Scale score is 7. Attempts at orotracheal intubation with restricition of cervical spinal motion are unsuccessful due to bleeding and distorted anatomy. The patient becomes apneic. The best procedure for airway management in this situation is: a. Nasotracheal intubation b. Emergency tracheostomy c. Surgical cricothyroidotomy d. Placement of an oropharyngeal airway e. Placement of an nasopharyngeal airway
22. A 25-year-old male is brought to the hospital after sustaining partial- and fullthickness burns involving 60% of his body surface area. General surgical services are available. His right arm and hand are severely burned. There are obvious fullthickness burns of the entire right hand and a circumferential burn of the right arm. Pulses are absent at the right wrist and are not detected by Doppler examination. Management should include: a. Fasciotomy b. Angiography c. Escharotomy d. Heparinization
Tangential excision
23. A 30-year-old male presents following a motor vehicle crash. Vital signs are respiratory rate 18, heart rate 88, blood pressure 130/72 mmHg, and Glascow Coma Scale score 13. Laparotomy is indicated when: a. There is a distinct seat-belt sign over the abdomen b. The CT scan demonstrates a grade 4 hepatic injury c. There is evidence of residual hemothorax after chest tube placement d. CT demonstrates retroperitoneal air e. CT demonstrates a retroperitoneal hematoma 24. All of the following are signs of aortic rupture on chest x-ray of a patient who sustained a blunt decelerating traume except which one? a. Mediastinal emphysema b. Presence of a pleural cap c. Obliteration of the aortic knob d. Deviation of the trachea to the right e. Depression of the left mainstem bronchus
25. A 30-year-old female is brought to the hospital after a fall from her bicycle. She was initially unconscious and then regained consciousness en route to the hospital. Thirty minutes after admission to the emergency department, her Glasoc Coma Scale score decreases to 6. her right pupil is larger than the left. She most likely has sustained: a. A subdural hemorrhage b. An epidural hemorrhage c. An occipital hemorrhage d. A subarachnoid hemorrhage e. An intracerebellar hemorrhage 26. You are treating a trauma patient and attempt a definitive airway by intubation. However, the vocal cords are not visibile. What tool would be the most valuable for achieving uccessful intubation? a. Gum elastic bougie b. Lateral cervical spine x-ray c. Nasopharyngeal airway d. Oxygen
e. Esophageal obturator airway
27. A 79-year old female is involved in a motor vehicle crash and presents to the emergency department. She is on warfarin (Coumadin®) and a beta blocker. Which of the following statements is true concerning her management? a. The risk of subdural hemorrhage is decreased. b. Absence of tachycardia indicates that the patient is hemodynamically normal. c. Nonoperative management of abdominal injuries is more likely to be successful in older adults than in younger patients. d. Vigorous fluid resuscitation may be associated with cardiopulmonary failure. e. Epinephrine should be infused immidiately for hypotension.
28. A 22-year-old male is brought by ambulance to a small community hospital after falling from the top of a 2.4-meter (8-foot) ladder. Initially, he was found to have a large right pneumothorax. A chest tube was inserted and connected to an underwater seal drainage collection system with negative pressure. A repeat AP portable chest xray demonstrates a residual, large, right pneumothorax. After transferring the patient to a verified trauma center, a third chest x-ray reveals a persistent right pneumothorax. The chest tube appears to be functioning and in good position. The patient remains hemodynamically normal with no signs of respiratory distress. The most likely cause for his persistent right pneumothorax is: a. Flail chest b. Diaphragmatic injury c. Pulmonary contusion d. Esophageal perforation e. Tracheobronchial injury
29. A 22-year-old female who is in the third trimester of pregnancy presents following a motor vehicle crash. Paramedics report vaginal bleeding. What is the next step in her treatment? a. Assess fetal heart sounds. b. Check for fetal movement. c. Perform inspection of the cervix. d. Ask the patient what her name is. e. Insert a wedge under the patient´s right hip.
30. A 40-year-old male is brought to the emergency department after a fall from a height of just over 3 meters (10 feet). His airway is clear, respiratory rate is 28, and systolic blood pressure is 140 mmHg. There is equal air entry on both sides of the chest, with comparable percussion sounds bilaterally. He complains of pain on palpation of the chest. Which intervention is most likely needed? a. Needle decompression of the chest b. Pericardiocentesis c. Pain management d. Thoracotomy e. Tube thoracostomy
31. A construction worker falls from a scaffold and is transferred to the emergency department. His heart rate is 124, and blood pressure is 85/60 mmHg. He complains of lower abdominal pain. After assessing the airway and chest, restricting cervical spinal motion, and initiating fluid resuscitation, the next step is to perform: a. FAST exam b. Detailed neurological exam c. Rectal exam d. Cervical spine x-ray e. Urethral catheterization
32. A 22-year-old male sustains a shotgun wound to the left shoulder and chest at close range. His blood pressure is 80/40 mmHG, and his heart rate is 130 beats per minute. Fluid resuscitation is initiated, his blood pressure increases to 122/84 mmHg, and heart rate decreases to 100 beats per minute. He is tachypneic, with a respiratory rate of 28 breaths per minute. On physical examination, his breath sounds are decreased at the left upper chest with dullness on percussion. A thoracostomy tube is inserted in the fifth intercostal space, with the return of 250 mL of blood and no air leak. The most appropriate next step is to:
a. Insert a Foley catheter b. Begin to transfuse O-negative blood c. Perform thoracotomy d. Obtain a CT scan of the chest and abdomen e. Repeat the physical examination of the chest
33. Which one of the following signs is associated with class II shock? a. Narrowed pulse pressure b. Urine output less than 15 mL per hour c. Respiratory rate above 35 breathes per minute d. Hypotension e. Obtundation 34. The most common acid-base disturbance encountered in injured pediatric patients is caused by: a. Hemorrhage b. Changes in ventilation c. Renal failure d. Injudicious bicarbonate administration e. Insufficient sodium chloride administration 35. Neurogenic shock is: a. Diagnosed by the presence of flaccid paralysis b. Caused by brain injury c. Due to acute hemorrhage d. Due to decreased vascular resistance e. Initially managed with vasopressor therapy 36. A 23-year-old male is admitted to the emergency department 3 hours after sustaining full-thickness burns to his head, arms, and upper torso for a total of 50% of his totalbody-surface area. He weighs 80 kg (185 pounds). His blood pressure is 105/75 mmHg, and his heart rate is 135 beats per minute. A urinary catheter is inserted, with the return of 20 mL of dark amber urine. He has received 1000 mL of Ringer´s lactate solution since the time of his injury. The estimated crystalloid fluid resuscitation volume per hour for the next 5 hours should be: a. 600 mL b. 875 mL c. 1000 mL d. 1200 mL e. 2000 mL 37. A 20-year-old male is brought to the hospital approximately 30 minutes after being stabbed in the chest. There is a 3-centimeter wound just medial to the left nipple. His blood pressure is 70/33 mmHg, and his heart rate is 140. Neck and arm veins are distended. Breath sounds are normal. Heart sounds are diminished. Which of the
following would be most useful in diagnosing and managing his injury? a. CT scan of the chest b. 12-lead ECG c. Left tube thoracostomy d. Initiation of vassopressors e. FAST exam 38. A 35-year-old motorcyclist is brought in after a frontal impact collision. His vital signs are heart rate 140, blood pressure 86/60 mmHg, and respiratory rate 36. Breath sounds are normal. He is complaining bitterly of lower abdominal pain. There seems to be a leg-length discrepancy and external rotation of the leg. Which one of the following statements concerning this patient is true? a. Pelvic injury can be ruled out based on the mechanism of injury. b. The patient most likely has a distal femur fracture. c. X-rays of the chest and pelvis are important in the initial evaluation. d. Impaired neurologic status of the left lower limb is expected. e. Prompt chest tube insertion should be considered. 39. A 17-year-old female is brought to the emergency department following a 2-meter (6-foot) fall onto concrete. She is unresponsive and found to have a respiratory rate of 32, blood pressure of 90/60 mmHg, and heart rate of 68. The first step in the treatment is: a. Administering vasopressors b. Establishing IV access for drug-assisted intubation c. Seeking the cause of her decreased level of consciousness d. Applying oxygen and maintaining her airway e. Excluding hemorrhage as a cause of shock 40. Which one of the following statements are true regarding diagnostic peritoneal lavage? a. DPL has no utility in the diagnosis of diaphragmatic ruptur. b. DPL should be performed whenever an indication for laparotomy is present. c. DPL has high specificity d. DPL can be used for diagnosing retroperitoneal injuries. e. DPL has a high sensitivity.
4 Diverse ATLS Post Test MCQ and Q&A 1. The primary indication for transferring a patient to a higher level trauma center is:
4. Which one of the following statements regarding patients with thoracic spine injuries is TRUE?
unavailability of a surgeon or operating room
Log-rolling may be destabilizing to
staff.
fractures from T-12 to L-1.
multiple system injuries, including severe head injury.
Adequate immobilization can be
accomplished with the scoop stretcher.
resource limitations as determined by the
Spinal cord injury below T-10 usually spares
bowel and bladder function.
transferring doctor.
resource limitations as determined by the hospital administration.
Hyperflexion fractures in the upper
thoracic spine are inherently unstable.
widened mediastinum on chest x-ray following blunt thoracic trauma.
These patients rarely present with spinal
shock in association with cord injury.
2. teen-aged bicycle rider is hit by b y a truck traveling at a high rate of speed. In the emergency department, she is actively bleeding from open fractures of her legs, and has abrasions on her chest and abdominal wall. Her blood pressure is 80/50 mm Hg, heart rate is 140 beats per minute, respiratory rate is 8 breaths per minute, and GCS score is 6. The first step in managing this patient is to:
5. young man sustains a ritle wound to the midabdomen. He is brought promptly to the emergency department by prehospital personnel. His skin is cool and diaphoretic, and his systolic blood pressure is 58 rnm Hg. Warmed crystalloid fluids are initiated without improvement in his vital signs. The next, most appropriate step is to perform: a celiotomy.
an abdominal CT scan.
obtain a lateral cervical spine x-ray.
diagnostic laparoscopy.
insert a central venous pressure line.
abdominal ultrasonography.
administer 2 liters of crystalloid solution.
a diagnostic peritoneal lavage.
perform endotracheal intubation and
ventilation.
apply the PASG and inflate the leg
compartments.
3. Contraindication to nasogastric intubation is the presence of a: gastric perforation.
diaphragmatic rupture.
open depressed skull fracture.
fracture of the cervical spine.
fracture of the cribriform plate.
6. young woman sustains a severe head he ad injury as the result of a motor vehicular crash. In the emergency department, her GCS score is 6. Her blood pressure is 140/90 mm Hg and her heart rate is 80 beats per minute. She is intubated and is being mechanically ventilated. Her pupils are 3 mm in size and equally equall y reactive to light. There is no other apparent injury. The most important principle to follow in the early management of her head injury is to: administer an osmotic diuretic.
prevent secondary brain injury.
aggressively treat systemic hypertension.
reduce metabolic requirements of the
brain. distinguish between intracranial hematoma
and cerebral edema.
7. 22-year-old man is brought to the hospital after crashing his motorcycle into a telephone pole. He is unconscious and in profound shock. He has no open wounds or obvious fractures. The cause of his shock is MOST LIKELY caused by:
9. 8-year-old girl is an unrestrained passenger in a vehicle struck from behind. In the emergency department, her blood pressure is 80/60 mm Hg, heart rate is 80 beats per minute, and respiratory rate is 16 breaths per minute. Her GCS score is 14. She complains that her legs feel "funny and won't move right;" however, her spine x-rays do not show a fracture or dislocation. A spinal cord injury in this child: is most likely a central cord syndrome.
must be diagnosed by magnetic resonance
imaging.
a subdural hematoma.
an epidural hematoma.
a transected lumbar spinal cord.
a transected cervical spinal cord.
findings on x-ray studies.
hemorrhage into the chest or abdomen.
is unlikely because of the incomplete
can be excluded by obtaining a CT of the
entire spine.
may exist in the absence of objective
calcification of the vertebral bodies.
8. 30-year-old man is struck by a car traveling at 56 kph (35 mph). He has obvious fractures of the left tibia near the knee, pain in the pelvic area, and severe dyspnea. His heart rate is 180 beats per minute, and his respiratory rate is 48 breaths per minute with no breath sounds heard in the left chest. A tension pneumothorax is relieved by immediate needle decompression and tube thoracostomy. Subsequently, his heart rate decreases to 140 beats per minute, his respiratory rate decreases to 36 breaths per minute, and his blood pressure is 80/50 inm Hg. Warmed Ringer's lactate is administered intravenously. The next priority should be to: perform a urethrogram and cystogram.
perform external fixation of the pelvis.
obtain abdominal and pelvic CT scans.
perform arterial embolization of the pelvic
vessels. perform diagnostic peritoneal lavage or abdominal ultrasound.
10. Immediate chest tube insertion is indicated for which of the following conditions? Pneumothorax
Pneumomediastinum
Massive hemothorax
Diaphragmatic rupture
Subcutaneous emphysema
11. 18-year-old, helmeted motorcyclist is brought by ambulance to the emergency department following a high-speed crash. Prehospital persormel report that he was thrown 15 meters (50 feet) off his bfice. He has a history of hypotension prior to arrival in the emergency department, but is now awake, alert, and conversational. Which of the following statements is TRUE? Cerebral perfiision is intacto
Intravascular volume status is normal.
The patient has sensitive vasomotor
reflexes.
Intraabdominal visceral injuries are
unlikely. The patient probably has an acute
epidural hematoma.
12. crosstable, lateral x-ray of the cervical spine:
administer 0-negative blood.
apply extemal warming devices.
control internal hemorrhage operatively.
apply the pneumatic antishock garment.
infuse large volumes of intravenous
crystalloid solution.
must precede endotracheal intubation.
excludes serious cervical spine injury.
is an essential part of the primary survey.
is not necessary for unconscious patients
with penetrating cervical injuries. is unacceptable unless 7 cervical vertebrae
defmitive treatment in managing this patient is to:
16. To establish a diagnosis of shock: systolic blood pressure must be below 90
mm Hg.
the presence of a closed head injury should
and the C-7 to T-1 relationship are
be excluded.
visualized.
acidosis should be present by arterial blood
\gas analysis. the patient must fail to respond to
13. During resuscitation, which one of the following is the most reliable as a guide to volume replacement?
intravenous fluid infu.sion. clinical evidence of inadequate organ
perfusion must be present.
Pulse rate
Hematocrit
Blood pressure
Urinary output
Jugular venous pressure
14. Which one of the following is the recommended method for initially treating frostbite? Vasodilators
Anticoagulants
Warm (40°C) water
Padding and elevation
Topical application of silvasulphadiazine
15. young man sustains a gunshot wound to the abdomen and is brought promptly to the emergency department by prehospital personnel. His skin is cool and diaphoretic, and he is confused. His pulse is thready and his femoral pulse is only weakly palpable. The
17. Absence of breath sounds and dullness to percussion over the left hemithorax are fmdings best explained by: left hemothorax.
cardiac contusion.
left simple pneumothorax.
left diaphragmatic rupture.
right tension pneumothorax.
18. 17-year-old helmeted motorcyclist is struck broadside by an automobile at an intersection. He is unconscious at the scene with a blood pressure of 140/90 mm Hg, heart rate of 90 beats per minute, and respiratory rate of 22 breaths per minute. His respirations are sonorous and deep. His GCS score is 6. Immobilization of the entire patient may include the use of all the following EXCEPT: air splints.
bolstering devices.
Young children are less tolerant of
a long spine board.
expanding intracranial mass lesions than
a scoop-style stretcher.
adults.
a semirigid cervical collar.
19. During an altercation, a 32-year-old man sustains a gunshot wound to the right upper hemithorax, above the nipple line with an exit wound posteriorly above the scapula on the right. He is transported by ambulance to a community hospital. He is endotracheally intubated, closed tube thoracostomy is performed, and 2 liters of Ringer's lactate solution are infused through 2 large-caliber IVs. His blood pressure now is 60/0 mm Hg, heart rate is 160 beats per minute, and respiratory rate is 14 breaths per minute (ventilated with 100% 02). The most appropriate next step in managing this patient is: celiotomy.
diagnostic peritoneal lavage.
arterial blood gas determination.
administer packed red blood cells.
chest x-ray to confinn tube placement.
20. Which of the following statements regarding iWhich of the following statements regardingnjury to the central nervous system in children is TRUE? Children suffer spinal cord injury without
21. 5-year-old boy is struck by an automobile and brought to the emergency department. He is lethargic, but withdraws purposefully from painful stimuli. His blood pressure is 90 mm Hg systolic, heart rate is 140 beats per minute, and his respiratory rate is 36 breaths per minute. The preferred route of venous access in this patient is: percutaneous femoral vein cannulation
cutdown on the saphenous vein at the
ankle. intraosseous catheter placement in the
proximal tibia. percutaneous peripheral veins in the
upper extremities.
central venous access via the subclavian or
interna1 jugular vein.
22. The response to catecholamines in an injured, hypovolemic pregnant woman can be expected to result in: placental abruption.
fetal hypoxia and distress.
fetal/maternal dysrhythmia.
improved uterine blood flow.
increased maternal renal blood flow.
x-ray abnormality more commonly than adults.
An infant with a traumatic brain injury may
become hypotensive from cerebral edema. Initial therapy for the child with traumatic
brain injury includes the administration of
result of traumatic brain injury when compared to adults .
is seldom life-threatening.
can be excluded by an upright, AP chest x-
ray. can be confused with a tension
methylprednisolone intravenously. Children have more focal mass lesions as a
23. Cardiac tamponade after trauma:
pneumothorax.
causes a fall in systolic pressure of > 15 mm
Hg with expiration. most commonly occurs after blunt injury to the anterior chest wall.
24. 30-year-old man sustains a severely comminuted, open distal right femur fracture in a motorcycle crash. The wound is actively bleeding. Normal sensation is present over the lateral aspect of the foot but decreased over the medial foot and great toe. Normal motion of the foot is observed. Dorsalis pedis and posterior tibial pulses are easily palpable on the left, but heard only by Doppler on the right. Immediate efforts to improve circulation to the injured extremity should involve: immediate angiography. tamponade of the wound with a pressure
fragments. realignment of the fracture segments with
a traction splint. fasciotomy of all four compartments in the
perform diagnostic peritoneal lavage.
initiate infusion of packed red blood cells.
insert a nasogastric tube and urinary
catheter. transfer the patient to the operating
room, while initiating fluid therapy.
initiate fluid therapy to return his blood
pressure to normotensive
dressing.
wound exploration and removal of bony
abdomen. Examination reveals a single bullet wound. He is breathing and has a thready pulse. However, he is unconscious and has no detectable blood pressure. Optimal immedi'ate management is to:
lower extremity.
25. 24-year-old man sustains multiple fractured ribs bilaterally as a result of being crushed in a press at a plywood factory. Examination in the emergency department reveals a flail segment of the patient's thorax. Primary resuscitation includes high-flow oxygen administration via a nonrebreathing mask, and initiation of Ringer' s lactate solution. The patient exhibits progressive confusion, cyanosis, and tachypnea. Management at this time should consist of: intravenous sedation. external stabilization of the chest wall. increasing the F102 in the inspired gas. intercostal nerve blocks for pain relief. endotracheal intubation and mechanical ventilation.
27. 25-year-old woman is brought to the emergency department after a motor vehicle crash. She was initially lucid at the scene and then developed a dilated pupil and contralateral extremity weakness. In t.he emergency department, she is unconscious and has a GCS score of 6. The initial management step for this patient should be to: obtain a CT scan of the head.
administer decadron 20 mg IV.
perform endotracheal intubation.
initiate an W line and administer Mannitol 1 g/kg. perform an emergency linar hole on t he
side of the dilated pupil.
28. Twenty-seven patients are seriously injured in an aircraft accident at a local airport. The basic principle of triage should be to: treat the most severely injured patients
first. establish a field triage area directed by a
doctor. rapidly transport all patients to the nearest
26. 23-year-old man is brought immediately to the emergency department from the hospital' s parking lot where he was shot in the lower
appropriate hospital. treat the greatest number of patients in the shortest period of time.
produce the greatest number of survivors
based on available resources.
29. Which one of the following statements is FALSE concerning Rh isoimmunization in the pregnant trauma patient? It occurs in blunt or penetrating abdominal
trauma. Minor degrees of fetomaternal
hemorrhage produce it. A negative Kleihauer-Betke test excludes
Rh isoimmunization. This is not a problem in the traumatized
Rh-positive pregnant patient. Initiation of Rh immunoglobulin therapy
inserting a nasopharyngeal airway.
performing a surgical cricothyroidotomy.
intubation. performing orotracheal intubation after
hemorrhage.
obtaining a lateral c-spine x-ray.
30. 24-year-old woman passenger in an automobile strikes the wind screen with her face during a head-on collision. In the emergency department, she is talking and has marked facial edema and crepitus. The highest priority should be given to: lateral, c-spine x-ray.
upper airway protection.
carotid pulse assessment.
management of blood loss.
determination of associated Injuries.
31. Early central venous pressure monitoring during fluid resuscitation in the emergency department has the greatest utility in a: patient with a splenic laceration.
patient with an inhalation injury.
6-year-old child with a pelvic fracture.
patient with a severe cardiac contusion.
hemothorax.
inserting an oropharyngeal airvvay.
performing fiberoptic-guided nasotracheal
does not require proof of fetomaternal
24-year-old man with a massive
32. 32-year-old man is brought to the hospital unconscious with severe facial injuries and noisy respirations after an automobile collision. In the emergency department, he has no apparent injury to the anterior aspect of his neck. He suddenly becomes apneic, and attempted ventilation with a face mask is unsuccessful. Examination of his mouth reveals a large hematoma of the pharynx with loss of normal anatomic landmarks. Initial management of his airway should consist of:
33. 42-year-old man, injured in a motor vehicle crash, suffers a closed head injury, multiple palpable left rib fractures, and bilateral femur fractures. He is intubated orotracheally without difficulty. Initially, his ventilations are easily assisted with a bag¬valve device. It becomes more difficult to ventilate the patient over the next 5 minutes, and his hemoglobin oxygen saturation level decreases from 98% to89 % . The most appropriate next step is to: obtain a chest x-ray.
decrease the tidal volume.
auscultate the patient's chest.
increase the rate of assisted ventilations.
perform needle decompression of the left
chest.
34. 42-year-old man is trapped from the waist down beneath his overtumed tractor for several hours before medical assistance arrives. He is awake and alert until just before arriving in the emergency department. He is now unconscious
and responds only to painful stimuli by moaning. His pupils are 3 mm in diameter and symmetrically reactive to light. Prehospital personnel indicate that they have not seen the patient move either of his lower extremities. On examination in the emergency department, no movement of his lower extremities is detected, even in response to painful stimuli. The most likely cause for this fmding is:
37. Regarding shock in the child, which of the following is FALSE? Vital signs are age-related.
Children have greater physiologic reserves
than do adults. Tachycardia is the primary physiologic response to hypovolemia. The absolute volume of blood loss
an epidural hematoma.
required to produce shock is the same as
a pelvic fracture.
in adults.
central cord syndrome.
An initial fluid bolus for resuscitation
intracerebral hemorrhage.
should approximate 20 mL/kg of Ringer's
bilateral compartment syndrome.
35. An electrician is electrocuted by a downed power line after a thunderstorm. He apparently made contact with the wire at the level of the right mid thigh. In the emergency department, his vital signs are normal and no dysrhythmia is noted on ECG. On examination, there is an exit wound on the bottom of the right foot. His urine is positive for blood by dip stick but no RBCs are seen microscopically. Initial management should include: immediate angiography.
aggressive fluid infusion.
intravenous pyleography.
debridement of necrotic muscle.
admission to the intensive care unit for
observation.
36. Which one of the following physical findings suggests a cause of hypotension other than spinal cord injury?
lactate.
38. All of the following signs on the chest x-ray of a blunt injury victim may suggest aortic rupture EXCEPT:: mediastinal emphysema.
presence of a "pleural cap."
obliteration of the aortic knob.
deviation of the trachea to the right.
depression of the left mainstem bronchus
39. An 8-year-old boy falls 4.5 meters (15 feet) from a tree and is brought to the emergency department by his family. His vital signs are normal, but he complains of left upper quadrant pain. An abdominal CT scan reveals a moderately severe laceration of the spleen. The receiving institution does not have 24-hour-aday operating room capabilities. The most appropriate management of this patient would be to: type and crossmatch for blood.
priapism.
request consultation of a pediatrician.
bradycardia.
transfer the patient to a trauma center.
diaphragmatic breathing.
admit the patient to the intensive care unit.
presence of deep tendon reflexes.
prepare the patient for surgery the next
day.
ability to flex forearms but inability to extend them.
b. 2L of IV crystalloid, manitol and IV steroids. c. 1 unit of albumin and compression stockings. d. Vassopressors and laprotomy. e. 2L of crystalloid and vassopressors if BPdoes not respond. 3. Which of the following signs is least reliable for diagnosing esophageal intubation?
1. A 20 year old athlete is involved in motorcycle crash. When he arrives in the ER. He shouts that he cannot move his legs. On physical exam, there are no abnormalities of the chest, abdomen, or
a. b. c. d. e.
Symmetrical chest wall movement. End-tidal CO2. Bilateral breath sounds. Oxygen saturation. Chest X-ray demonstrating the ETT tip positioned above the carnia.
pelvis. The patient has no sensation in his legs and cannot move them, but his arms are moving. The patient's respiratory rate is 23, heart rate 88, and blood pressure is 80/60 mm Hg. He is pale and sweaty. What is most likely cause of his condition?
a. b. c. d. e.
Neurogenic shock. Cardiac temponade. Myocardial contusion. Hypothermia. Abdominal hemorrhage.
2. A 22 year old male presents following a motorcycle crash. He complains of the inability to move or feel his legs. His blood pressure is 80/50 mm Hg, heart rate is 70, respiratory rate is 18. GCS is 15. Oxygen is 99%on 2L nasal prongs. Chest X-ray, pelvic X-ray, FAST are normal. Extremities are normal. His management should be: a. 2L of IV crystalloid and two units of pRBSs.
4. A 6 months old infant, being held in her mother's arms, is ejected on impact from a vehicle that is struck head-on by an oncoming car travelling at 64 kph. The infant arrives in the ER with multiple facial injuries, is lethargic, and in severe respiratory distress. Respiratory support is not effective using a bag mask device. And her oxygen saturation is falling. Repeated attempts at orotracheal intubation are unsuccessful. The most appropriate procedure to perform next is: a. b. c. d. e.
Administer hellox and epinephrine. Perform nasotracheal intubation. Perform surgical cricothyriodotomy. Repeat orotracheal intubation. Perform needle cricothyroidtomy with jet insufflations.
5. 28 year old male is brought to ER. He was involved in fight, in which he was
beaten with a wooden stick. His chest shows multiple severe bruises. Airway is clear, respiratory rate is 22, heart rate is 126, and systolic blood pressure is 90 mm Hg. Which of the following should be performed during the primary survey? a. b. c. d. e.
GCS. Tetanus toxoid administration. Cervical spine X-ray. Blood alcohol level. Rectal exam.
6. Which one of the following injuries is addressed in the secondary survey? a. b. c. d. e.
Forearm fracture. Mid-thigh amputation. Open fracture with bleeding. Unstable pelvic fracture. Bilateral femur fractures with obvious deformity.
7. Which one of the following statements is true regarding access in pediatric resuscitation? a. Intraosseous access should only be considered after five percutaneous attempts. b. Cut-down at the ankle is the preferred initial access technique. c. Internal jugular cannulation is the next preferred option when percutaneous venous access fails. d. Intraosseouscannulation should be the first choice for access. e. Blood transfusion can be delivered through intraosseous access.
8. A 35 year old female sustains multiple injuries in a motor vehicle crash and is transported to a small hospital in full spinal protection. She has a GCS of 4 and is being mechanically ventilated. Intravenous access is established and warmed crystalloid is infused. She remains hemodynamically normal and full spinal protection in maintained. Preparations are made to transfer her to another facility for definitive neurosurgical care. Prior to transport, which of the following tests or treatments is mandatory? a. FAST exam. b. Chest X-ray. ??? c. Lateral cervical spine X-ray.??? d. Administration of methiprednisolone. e. CT of abdomen. 9. A 23 year old male is stabbed below the right nipple. He is alert, and his ox ygen is 98%. Chest tube was placed for treatment of hemopnueunthorax. BP 90/60 mm Hg after 1L of crystalloid solution. What is the next step in treatment? a. b. c. d.
Place a left-sided chest tube. Re-examine the chest. Inscert central venous catheter. Perform CT scan of the abdomen and pelvis. e. Prepare for urgent throacotomy. 10. A 22 year old male is assaulted in a bar. A semi-rigid cervical coller is applied, and he is immobilized on a spine board. On initial exam, VS are normal, GCS is 15. Which of the following is an
indication for CT in this patient with possible minor traumatic brain injury?
following statements concerning this patient is true?
a. Presence of hemotympanum. b. Blood alcohol concentration of 0.16%. c. Presence of an isolated 10 cm scalp laceration. d. Presence of a mandibuler fracture. e. History of assult.
a. Fetal assessment should take priority. b. Log rolling the patient to the right will decompress the vena cava. c. Rh-immunoglubulin therapy should be immediately administered. d. The patient has likely impending respiratory failure. e. Vasopressors should be given to the patient.
11. A 23 year old construction worker is brought to ER after falling more than 9 meters. VS: HR is 140, BP is 90/60, and RR is 36. He is complaining bitterly of lower abdominal and lower limb pain, and his obvious deformity of both lower legs with bilateral open tibial fractures. Which of the following statements concerning the patient is true? a. Pelvic injury can be ruled out based on the mechanism of injury. b. Blood loss from the lower limbs is the most likely cause of hypotension. c. X-ray of the chest and pelvis are important adjacent in his initial assessment. d. Spinal cord injury is most likely cause of his hypotension. e. Aortic injury is the most likely cause of his tachycardia. 12. A 25 year old female in the third trimester of pregnancy is brought to ER following a high-speed motor vehicle crash. She is conscious and immobilized on long spine board. RR is 24, HR is 120, and BP is 70/50. Labs show a PaCO2 of 40 mm Hg. Which one of the
13. A 30 year old male is stabbed in the right chest. On arrival to ER, he is very short of breath. HR is 120 and BP is 80/50 mm Hg. His neck veins are flat. There is no diminished air entry on the right side, and there is dullness posteriorly on percussion. These findings are most consistent with: a. b. c. d. e.
Tension pneumothorax. Pericardial tamponade. Hypovolemia from liver injury. Hemothorax. Spinal cord injury.
14. A specific aspect of the treatment of thermal injuries is: a. Chemical burns require the immediate removal of clothing. b. Patients who sustain thermal injury are at lower risk of hypothermia. c. Patients with circumferential burns need prompt fasciotomies. d. Electrical burns are associated with extensive skin necrosis (from entry point to exit).
e. The Parkland formula should be used to determine adequacy of resuscitation. 15. A 15 year old male is brought to ER after being involved in a motor vehicle crash. He is unconscious and was intubated at the scene by EMS. On ER, O2 is 92%, HR is 96 and BP is 150/85 mm Hg. Breath sounds are decreased on the left side of the thorax. The next step is: a. Immediate needle cricothyroidotomy. b. Immediate needle thoracocentesis. c. Chest tube insertion. d. Reassess the position of endotracheal tube. e. Obtain a chest X-ray. 16. Which one of the following statements is true? a. Elevated ICP will not affect cerebral perfusion. b. CSF cannot be displaced from the cranial vault. c. Cerebral blood flow (CBF) is increased when the PaCO2 is below 30 mm Hg. d. AutoregulationCBF normally occurs between cerebral perfusion pressures of 50 to 150. e. Hypotonic fluids should be used to limit brain edema in patients with severe head injury. 17. The first priority in the management of a long bone fracture is: a. Reduction of the pain.
b. Prevention of infection in case of an open fracture. c. Prevention of further soft tissue injury. d. Control of hemorrhage. e. Improve long-term function. 18. A 40 year old obese patient with GCS of 8 requires a CT. before transfer to CT you should: a. Give more sedative drugs. b. Insert a multilumen esophageal airway. c. Insert a definitive airway. d. Request a lateral cervical spine film. e. Insert a nasogastric tube. 19. Lateral cervical spine films: a. Must be performed in the primary survey. b. Can exclude any significant spinal injury. c. Should be combined with clinical exam, AP and odontoid, CT. d. Are indicated in all trauma patients. e. Require the following films: oblique views, AP, odontoid and flexionextension views prior to spinal clearance in trauma patients. 20. A 30 year old male is brought to ER after falling 6 m. Flail chest on the right, tachypneic and normal breath sounds. No hyperresonance or dullness. On oxygen by face mask.ABG are: PaO2 of 45, PaCO2 of 28 and pH of 7.47. Abnormalities in the patient's blood gases is due to: a. Hypoventilation.
b. c. d. e.
Hypovolemia. Small pneumothorax. Pulmonary contusion. Flail chest.
21. An 82 year old male falls down five stairs and presents to the ER. All are true statements regarding his condition compared to a younger patient with similar mechanism, except: a. He is more likely to have had contracted circulatory volume prior to his injury. b. His risk of cervical spine injury is increased due to degeneration, stenosis, and loss of disk compressibility. c. Intracranial hemorrhage will become sympotmatic more quickly. d. His risk of occult fractures is increased. e. His risk of bleeding may be increased. 22. The most specific test to evaluate for injuries of solid abdominal organs is: a. b. c. d. e.
Abdominal X-ray. Abdominal U/S. DPL Frequent abdominal examination CT of abdomen and pelvis.
23. A 14 year old female is brought to ER after falling from a horse. She is immobolizedon a long spine board with a hard collar and blocks, cervical spine X-rays: a. Will show cervical spine injury in more than 20% of these patients.
b. Will exclude cervical spine injury if no abnormalities are found on the Xrays. c. Are not needed if she is awake, alert, neurologically normal, and has no neck pain or midline tenderness. d. Should be performed before addressing potential breathing or circulatory problems. e. May show atlanto-occipital dislocation if the power's ratio is < 1. 24. The most important consequences of inadequate organ perfusion is: a. b. c. d. e.
Multiple organ failure. Decreased base deficit. Acute gloumerulnephritis. Increased ATP production. Vasodilatation.
25. Hypertension following a head injury: a. Should be treated to reduce ICP. b. May indicate imminent herniation from critically high ICP. c. Indicates pre-existing hypertension. d. Mandates prompt administration of mannitol. e. Should prompt burr hole drainage of potential subdural hematomas. 26. Initial treatment of frostbite injuries involves: a. Application of dry heat. b. Rapid rewarming of the body part in circulating WARM water. c. Debridement of hemorrhagic blisters. d. Early amputation to prevent septic complications.
e. Massage of the affected area. 27. Signs and symptoms of airway compromise include all of the following except: a. b. c. d. e.
Change in voice. Stridor. Decreased pulse pressure. Dyspnea and agitation. Tachypnea.
28. Which one of the following statements is true regarding a pregnant patient who presents following blunt trauma? a. Early gastric decompression is important. b. A hemoglobin level of 10 g/dl indicates recent blood loss. c. The central venous pressure response to volume resuscitation is blunted in pregnant patients. d. A lap belt is the best form of restraint due to the size of the gravid uterus. e. A PaCO2 of 40 mm Hg provides reassurance about the adequacy of respiratory function. 29. A 30 year old is brought to ER after being injured in a motor vehicle crash. BP is 90/60 and HR is 122. She responds to the rapid infusion of 1 liter of crystalloid solution with a rise in her BP to 118/ 88 and a decrease in her HR to 90. Her BP then suddenly decreases to 96/66. The least likely cause of her hemodynamic change is: a. Traumatic brain injury. b. Ongoing blood loss.
c. Blunt cardiac injury. d. Inadequate resuscitation. e. Tension pneumothorax. 30. Limb-threatening extremity injuries: a. Require a tourniquet. b. Are characterized by the presence of ischemic or crushed tissue. c. Should be definitively managed by application of a traction splint. d. Are rarely present without an open wound. e. Indicates a different order of priorities for the patient's initial assessment and resuscitation. 31. A 29 year old female arrives in ER after being involved in a motor vehicle crash. She is 30 weeks pregnant. She was restrained with a lap and shoulder belt, and an airbag deployed. Which one of the following statements best describes the risk of injury? a. The deployment of the airbag increases the risk of fetal loss. b. The use of seatbelts is associated with increased risk of maternal death. c. The mechanism of injury suggests the need for emergency C-section due to the risk of impending abruption placenta. d. The risk of premature fetal delivery and death is reduced by the use of restraints. e. The deployment of the airbag increases the risk of maternal abdominal injury. 32. Supraglottic airway devices:
a. Are equivalent to endotracheal intubation. b. Require neck extension for proper placement. c. Are preferable to endotracheal intubation in a patient who cannot lie flat. d. Are of value as part of a difficult or failed intubation plan. e. Provide one form of definitive airway. 33. A 70 year old male suffers blunt chest trauma after being struck by a car. On ER, GCS is 15, BP is 145/90, HR is 72, RR is 24 and O2 saturation on 5L is 91%. Chest X-ray shows multiple rib fractures. ECG shows normal sinus rhythm with no conduction abnormalities. Management should include: a. Placement of a 22-french, rightsided chest tube. b. Serial troponins and cardiac monitoring. c. Thoracic splinting, taping, and immobilization. d. Monitored IV analgesia. e. Bronchoscopy to exclude tracheobronchial injury. 34. A 15 year old male presenting after a motorcycle crash. Initial exam reveals normal VS. There is a large bruise over his epigastrium that extends to left flank. He has no other appearent injuries. A CT of abdomen shows ruptured spleen surrounded by a large hematoma and fluis in the pelvis. The
next step in this patient's management is: a. b. c. d. e.
Splenic artery immobilization. Pneumococcal vaccine. Transfer to pediatrician. Urgent laparotomy. Surgical consultation.
35. A 30 year old male presents with a stab wound to the abdomen. BP is 85/60, HR is 130, RR is 25 and GCS is 14. Neck veins are flat, and chest exam is clear with bilateral breath sounds. Optimal resuscitation should include: a. Transfusion of fresh frozen plasma and platelets. b. 500 mL of hypertonic saline and transfusion of pRBSs. c. Resuscitation with crystalloid and pRBCs until base excess is normal. d. Preparation for laparotmy while initiating fluid resuscitation. e. Fluid resuscitation and angioemobolization. 36. Initial resuscitation in adult trauma patients should: a. Be with 1-2 liters of crystalloid, monitoring the patient's response. b. Use crystalloid to normalize BP. c. Use permissive hypotension in patients with head injury. d. Be with anon-blood colloid solution. e. Be a minimum of 2 L of crystalloid in all trauma patients prior to administering blood. 37. A 25 year old male is brought to ER following a bar fight. He has an altered
level of consciousness, opens his eyes on command, moans without forming discernible words, and localizes to painful stimuli. Which one of the following concerning this patient is correct? a. Mandatory intubation to protect his airway is required. b. His GCS suggests a severe head injury. c. His level of consciousness can be solely attributed to elevated blood alcohol. d. CT is an important part of neurological exam. e. Hyperoxia should be avoided.
c. Distended neck veins. d. Diaphragmatic breathing. e. Ability to flex forearms but inability to extend them. 40. Cardiac temponade: a. Is definitively managed by needle pericardiocentesis. b. Is most common with blunt thoracic trauma and anterior rib fractures. c. Is easily diagnosed by discovery of Beck's triad in the ER. d. Is indicated by Kussmaul breathing. e. Requires surgical intervention.
38. Which one of the following statements regarding genitourinary injuries is true? a. Urethral injuries are associated with pelvic fractures. b. All patients with microscopic hematuria require evaluation of the genitourinary tract. c. Patients with gross hematuria and shock will have a major renal injury as the source of hemorrhage. d. Intraperitoneal bladder injuries are usually managed definitively with a urinary catheter. e. Urinary catheters should be placed in all patients with pelvic fractures during the primary survey. 39. Which one of the following physical findings does not suggest spinal cord injury as the cause of hypotension? a. Priapism. b. Bradycardia.
First group ATLS questions.. 1- triage concept : save more lives with available resources 2- a patient with gun shot , BP 70/0 , Chest tube drained 120 ml , chest sounds normal. next step? - Laboratomy 3- persistent pneumothorax after placing chest tube. Diagnosis? - Tracheobronchial injury 4- which of the following is not part of the initial assessment? - determining incomplete,or complete neurological deficit 5- echomosys in prenium , blood in mayatus ,what will you do? - retrograde urethrogram 6- Class II shock: - normal BP
7- patient with minimal trauma to chest and tenderness,ABCDE are good, how to manage? - pain medication 8- old patient on B-blocker and Coumadin (warfarin), decreased BP, normal pulse, which of the following is true? - excessive fluids cause cardiopulmonary failure. 9- indication for intubation? - maxilofacial injury 10- moderate head injury- GCS 11, what to do? - CT scan+repeat GCS assessment 11-Neurogenic shock except: neurologic deficit 12- Neurogenic shock management: vassopressor 13- initial step in multiple injured pt: ask the patient's name 14- pt. with blunt trauma to abdomen.. Decreased BP, no external bleeding: FAST 15- DPL: most sensitive 16- unsucceful endotracheal intubation: surgical cricothyroidectomy 17- anatomy: choroid plexus produce CSF in the lateral and third ventricle.. 18- failed intubation: bogie 19- unconscious baby then awake then deteriorate: epidural 20- pt fell and can't move lower limb with sensory problem: spinal shock 21- laparotomy indication: CT with retroperitoneal air 22- circumfrential burn: escharotomy 23-Pediatric: flexible mediastinum 24- sacral sparing: good prognosis 25- most common cause of acid base problem in pediatrics: ventilation 26- 80 kg male with 50% burn and received 1 L NS came after 3 hour.. What is the fluid per hour in the next 5 hours?1400 ml/hr
27- X-ray of ruptured aorta except: air in mediastinum 28- indication to anti-Rh in pregnant women except: gunshot wound to the foot 29- pregnant lady with PV bleeding, initial assessment Airway.. 30- difficulty in respiration, loss of sensation in the foot?- possible cervical injury
1. Which of the following is addressed in the secondary survey? Answer: Forearm fracture 2. A young male fallen from height with obvious flail chest. ABG shows pH of 7.47. what is the cause of this abnormality? Answer: pulmonary contusion 3. Cushing's triad which occurs in cases of increased intracranial pressure? Answer: Bradycardia with irregular respirations and isolated increase in SYSTOLIC BLOOD PRESSURE. 4. Trauma in pregnant women, clear fluid leakage from vagina is an indication for hospitalization. 5. blood at the external meatus do RUG 6. Scenario of pelvic fracture, which statement is correct? Answer: vertical shear force with posterior ligaments disruption. 7. 12 year old boy sustained blunt abdominal trauma while playing football. FAST scan is positive. He is hemodynamically stable. What to do next? Answer: CT abdomen
8. Old patient, had multiple rib fractures, splinting of the right chest, what to do? Answer: give analgesic. 9. Memorize indications for burn transfer ( eg> 10% surface area affected transfer) 10. In comparison with young adults, elderly patients exhibit which of the following regarding brain injuries? Aincreased cerebral blood flow. B- less stretching of bridging veins C- less subdural hematomas D- less brain contusions E- les mobility with angular acceleration and deceleration. 11. Which of the following will be missed by DPL? Answer: subcapsular hematoma of the spleen (because it is a retroperitoneal organ) 12. Which of the following tests will evaluate the retroperitonium? Answer: CT 13. Patient tried to commit suicide with a rope (hyperextension of the neck), when he presented to ER he had hoarsness of voice and crepitation in the neck, what to do? A- needle cricothyroidotomy Bsurgical tracheostomy in OR C- direct laryngoscopy and intubation. Answer: ???? 14. Burn victim with signs of inhalation injury (carboneacous material, singed eyebrows) Intubate. 15. Burn victim, has circumferential burn, core temperature is 34 C. what next? Aescharatomy B- rewarm C- oxygen mask (I think the answer is C oxygen mask because you should follow the ABCD priorities) 16. Which of the followings is NORMAL in pregnancy? A-increased residual lung volume B- decreased plasma volume C-
decreased total RBC mass D- widened symphysis pubis. Answer is D 17. Trauma patient was hypotensive then you gave him 1 L of crystalloid and now he is alert and talking. Which of the following statements is correct? He has good cerebral blood flow 18. Which of the followings is a contraindication for nasal intubation? Adepressed skull fracture B- Le Fort III fractures. Answer is B (Le Fort III fractures causes cribriform plate fracture) 19. Which of the followings will benefit from oropharyngeal airway? Aposterior displacement of tongue Blaryngeal edema. Answer is A 20. Patient came with severe head injury GCS is 6 and has poor anal sphincter tone and diagrammatic breathing. His hands are flexed across the chest. What is the cause of his injury? A-isolated head injury. B-lumbosacral verterbral injury. C- Cervical vertebral injury. Answer is C 21. Question about widened mediastinum 22. Question about spinal board? Remove after completing assessment or it leads to decubetus ulcers .. 23. CSF is between? Arachinoid and pia matter. 24. Question about transfer? Transfer after stabilizing the patient. 25. Scenario with scalp laceration .. the priority was to stop the bleeding with direct pressure. The other choices are all after the circulation. 26. Patient with head injury and systemic hypotension, what is the most probable cause of his hypotension? A- bleeding in the brain B- systemic hypovolemia. Answer is B
27. Dorsal column? Carries proprioception from the same side of the body.
SOAL POST TEST 1. Cardiac tamponade after trauma a. is seldom life-threatening b. can be excluded by an upright, AP chest x-ray c. can be confused with a tension pneumothorax d. causes a fall in systolic pressure of > 15 mm Hg with expiration e. most commonly occurs after blunt injury to the anterior chest wall 2. Which one of the following statements regarding patients with thoracic spine injuries is TRUE? a. Log-rolling may be destabilizing to fractures from T-12 to L-1. b. Adequate immobilization can be accomplished with the scoop stretcher. c. Spinal cord injury below T-10 usually spares bowel and bladder function. d. Hyperflexion fractures in the upper thoracic spine are inherently unstable. e. These patients rarely present with spinal shock in association with cord injury. 3. Absence of breath sounds and dullness to percussion over the left hemithorax are fmdings best explained by a. Left hemothorax. b. c. d. e. f. g. h.
cardiac contusion left simple pneumothorax
i.
left diaphragmatic rupture
j.
right tension pneumothorax.
4. A young man sustains a gunshot wound to the abdomen and is brought promptly to the emergency department by prehospital personnel. His skin is cool and diaphoretic, and he is confused. His pulse is thready and his femoral
pulse is only weakly palpable. The defmitive treatment in managing this patient is to a. administer 0-negative blood b. applyextemal warming devices. c. Control internal hemorrhage operatively d. apply the pneumatic antishock garment e. infuse large volumes of intravenous crystalloid solution. 5. To establish a diagnosis of shock, a. systolic blood pressure must be below 90 mm Hg. b. the presence of a closed head injury should be excluded c. acidosis should be present by arterial blood \gas analysis d. the patient must fail to respond to intravenous fluid infusion. e. clinical evidence of inadequate organ perfusion must be present. 6. A 23-year-old man is brought immediately to the emergency department from the hospital' s parking lot where he was shot in the lower abdomen. Examination reveals a single bullet wound. He is breathing and has a thready pulse. However, he is unconscious and has no detectable blood pressure. Optimal immediate management is to a. perform diagnostic peritoneal lavage. b. initiate infusion of packed red blood cells. c. insert a nasogastric tube and urinary catheter. d. transfer the patient to the operating room, while initiating fluid therapy. e. initiate fluid therapy to return his blood pressure to normotensive 7. An electrician is electrocuted by a downed power line after a thunderstorm. He apparently made contact with the wire at the level of the right mid thigh. In the emergency department, his vital signs are normal and no dysrhythmia is noted on ECG. On examination, there is an exit wound on the bottom of the right foot. His urine is positive for blood by dip stick but no RBCs are seen microscopically. Initial management should include a. immediate angiography.
b. aggressive fluid infusion. c. intravenouspyleography. d. debridement of necrotic muscle. e. admission to the intensive care unit for observation.
8. An 8-year-old girl is an unrestrained passenger in a vehicle struck from behind. In the emergency department, her blood pressure is 80/60 mm Hg, heart rate is 80 beats per minute, and respiratory rate is 16 breaths per minute. Her GCS score is 14. She complains that her legs feel "funny and won't move right;" however, her spine x-rays do not show a fracture or dislocation. A spinal cord injury in this child a. is most likely a central cord syndrome. b. must be diagnosed by magnetic resonance imaging. c. can be excluded by obtaining a CT of the entire spine. d. may exist in the absence of objective findings on x-ray studies. e. is unlikely because of the incomplete calcification of the vertebral bodies. 9. Immediate chest tube insertion is indicated for which of the following conditions? a. Pneumothorax b. Pneumomediastinum c. Massive hemothorax d. Diaphragmatic rupture e. Subcutaneous emphysema 10.A 32-year-old man is brought to the hospital unconscious with severe facial injuries and noisy respirations after an automobile collision. In the emergency department, he has no apparent injury to the anterior aspect of his neck. He suddenly becomes apneic, and attempted ventilation with a face mask is unsuccessful. Examination of his mouth reveals a large hematoma of the
pharynx with loss of normal anatomic landmarks. Initial management of his airway should consist of a. inserting an oropharyngealairvvay. b. inserting a nasopharyngeal airway. c. performing a surgical cricothyroidotomy. d. performingfiberoptic-guided nasotracheal intubation. e. performingorotracheal intubation after obtaining a lateral c-spine x-ray.
11.The primary indication for transferring a patient to a higher level trauma center is a. unavailability of a surgeon or operating room staff. b. multiple system injuries, including severe head injury. c. resource limitations as determined by the transferring doctor. d. resource limitations as determined by the hospital administration. e. widened mediastinum on chest x-ray following blunt thoracic trauma. 12.A young man sustains a ritle wound to the mid-abdomen. He is brought promptly to the emergency department by prehospital personnel. His skin is cool and diaphoretic, and his systolic blood pressure is 58 rnm Hg. Warmed crystalloid fluids are initiated without improvement in his vital signs. The next, most appropriate step is to perform a. celiotomy. b. an abdominal CT scan. c. diagnostic laparoscopy. d. abdominal ultrasonography. e. a diagnostic peritoneal lavage. 13.A teen-aged bicycle rider is hit by a truck traveling at a high rate of speed. In the emergency department, she is actively bleeding from open fractures of her legs, and has abrasions on her chest and abdominal wall. Her blood
pressure is 80/50 mm Hg, heart rate is 140 beats per minute, respiratory rate is 8 breaths per minute, and GCS score is 6. The first step in managing this patient is to a. obtain a lateral cervical spine x-ray. b. insert a central venous pressure line. c. administer 2 liters of crystalloid solution. d. perform endotracheal intubation and ventilation. e. apply the PASG and inflate the leg compartments. 14.An 8-year-old boy falls 4.5 meters (15 feet) from a tree and is brought to the emergency department by his family. His vital signs are normal, but he complains of left upper quadrant pain. An abdominal CT scan reveals a moderately severe laceration of the spleen. The receiving institution does not have 24-hour-a-day operating room capabilities. The most appropriate management of this patient would be to a. type and crossmatch for blood. b. request consultation of a pediatrician. c. transfer the patient to a trauma center. d. admit the patient to the intensive care unit. e. prepare the patient for surgery the next day. 15.A 17-year-old helmeted motorcyclist is struck broadside by an automobile at an intersection. He is unconscious at the scene with a blood pressure of 140/90 mm Hg, heart rate of 90 beats per minute, and respiratory rate of 22 breaths per minute. His respirations are sonorous and deep. His GCS score is 6. Immobilization of the entire patient may include the use of all the following EXCEPT a. air splints. b. bolstering devices. c. a long spine board. d. a scoop-style stretcher. e. A semirigid cervical collar.
16.Which of the following statements regarding injury to the central nervous system in children is TRUE? a. Children suffer spinal cord injury without x-ray abnormality more commonly than adults. b. An infant with a traumatic brain injury may become hypotensive from cerebral edema. c. Initial therapy for the child with traumatic brain injury includes the administration of methylprednisolone intravenously. d. Children have more focal mass lesions as a result of traumatic brain injury when compared to adults . e. Young children are less tolerant of expanding intracranial mass lesions than adults.
17.During an altercation, a 32-year-old man sustains a gunshot wound to the right upper hemithorax, above the nipple line with an exit wound posteriorly above the scapula on the right. He is transported by ambulance to a community hospital. He is endotracheally intubated, closed tube thoracostomy is performed, and 2 liters of Ringer's lactate solution are infused through 2 large-caliber IVs. His blood pressure now is 60/0 mm Hg, heart rate is 160 beats per minute, and respiratory rate is 14 breaths per minute (ventilated with 100% 02). The most appropriate next step in managing this patient is a. celiotomy. b. diagnostic peritoneal lavage. c. arterial blood gas determination. d. administer packed red blood cells. e. chest x-ray to confinn tube placement. 18.A 42-year-old man, injured in a motor vehicle crash, suffers a closed head injury, multiple palpable left rib fractures, and bilateral femur fractures. He is intubated orotracheally without difficulty. Initially, his ventilations are easily assisted with a bag¬valve device. It becomes more difficult to ventilate the
patient over the next 5 minutes, and his hemoglobin oxygen saturation level decreases from 98% to89 % . The most appropriate next step is to a. obtain a chest x-ray. b. decrease the tidal volume. c. auscultate the patient's chest. d. increase the rate of assisted ventilations. e. perform needle decompression of the left chest. 19.A 24-year-old woman passenger in an automobile strikes the wind screen with her face during a head-on collision. In the emergency department, she is talking and has marked facial edema and crepitus. The highest priority should be given to a. lateral, c-spine x-ray. b. upper airway protection. c. carotid pulse assessment. d. management of blood loss. e. determination of associated Injuries. 20.Twenty-seven patients are seriously injured in an aircraft accident at a local airport. The basic principle of triage should be to a. treat the most severely injured patients first. b. establish a field triage area directed by a doctor. c. rapidly transport all patients to the nearest appropriate hospital. d. treat the greatest number of patients in the shortest period of time. e. produce the greatest number of survivors based on available resources. 21.Which one of the following statements is FALSE concerning Rh isoimmunization in the pregnant trauma patient? a. It occurs in blunt or penetrating abdominal trauma. b. Minor degrees of fetomaternal hemorrhage produce it. c. A negative Kleihauer-Betke test excludes Rh isoimmunization. d. This is not a problem in the traumatized Rh-positive pregnant patient.
e. initiation of Rh immunoglobulin therapy does not require proof of fetomaternal hemorrhage. 22.A 30-year-old man is struck by a car traveling at 56 kph (35 mph). He has obvious fractures of the left tibia near the knee, pain in the pelvic area, and severe dyspnea. His heart rate is 180 beats per minute, and his respiratory rate is 48 breaths per minute with no breath sounds heard in the left chest. A tension pneumothorax is relieved by immediate needle decompression and tube thoracostomy. Subsequently, his heart rate decreases to 140 beats per minute, his respiratory rate decreases to 36 breaths per minute, and his blood pressure is 80/50 inm Hg. Warmed Ringer's lactate is administered intravenously. The next priority should be to: a. perform a urethrogram and cystogram. b. perform external fixation of the pelvis. c. obtain abdominal and pelvic CT scans. d. perform arterial embolization of the pelvic vessels. e. perform diagnostic peritoneal lavage or abdominal ultrasound. 23.Regarding shock in the child, which of the following is FALSE? a. Vital signs are age-related. b. Children have greater physiologic reserves than do adults. c. Tachycardia is the primary physiologic response to hypovolemia. d. The absolute volume of blood loss required to produce shock is the same as in adults. e. An initial fluid bolus for resuscitation should approximate 20 mL/kg of Ringer's lactate 24.A 30-year-old man sustains a severely comminuted, open distal right femur fracture in a motorcycle crash. The wound is actively bleeding. Normal sensation is present over the lateral aspect of the foot but decreased over the medial foot and great toe. Normal motion of the foot is observed. Dorsalispedis and posterior tibial pulses are easily palpable on the left, but
heard only by Doppler on the right. Immediate efforts to improve circulation to the injured extremity should involve a. immediate angiography. b. tamponade of the wound with a pressure dressing. c. wound exploration and removal of bony fragments. d. realignment of the fracture segments with a traction splint. e. fasciotomy of all four compartments in the lower extremity. 25.A crosstable, lateral x-ray of the cervical spine a. must precede endotracheal intubation. b. excludes serious cervical spine injury. c. is an essential part of the primary survey. d. is not necessary for unconscious patients with penetrating cervical injuries. e. is unacceptable unless 7 cervical vertebrae and the C-7 to T-1 relationship are visualized.
26.An 18-year-old, helmeted motorcyclist is brought by ambulance to the emergency department following a high-speed crash. Prehospitalpersormel report that he was thrown 15 meters (50 feet) off his bfice. He has a history of hypotension prior to arrival in the emergency department, but is now awake, alert, and conversational. Which of the following statements is TRUE? a. Cerebral perfiision is intact b. Intravascular volume status is normal. c. The patient has sensitive vasomotor reflexes. d. Intraabdominal visceral injuries are unlikely. e. The patient probably has an acute epidural hematoma.
27.Which one of the following is the recommended method for initially treating frostbite? a. Vasodilators b. Anticoagulants c. Warm (40°C) water d. Padding and elevation e. Topical application of silvasulphadiazine 28.The driver of a single car crash is orotracheally intubated in the field by prehospital personnel after they identify a closed head injury and determine that the patient is unable to protect his airway. In the emergency department, the patient demonstrates decorticate posturing bilaterally. He is being ventilated with a bag-valve device, but his breath sounds are absent in the left hemithorax. His blood pressure is 160/88 mm Hg, heart rate is 70 beats per minute, and the pulse oximeter displays a hemoglobin oxygen saturation of 96% . The next step in assessing and managing this patient should be to a. determine the arterial blood gases. b. obtain a lateral cervical spine x-ray. c. assess placement of the endotracheal tube. d. perform needle decompression of the left chest. e. insert a thoracostomy tube in the left hemithorax. 29.Early central venous pressure monitoring during fluid resuscitation in the emergency department has the greatest utility in a a. patient with a splenic laceration. b. patient with an inhalation injury. c. 6-year-old child with a pelvic fracture. d. patient with a severe cardiac contusion. e. 24-year-old man with a massive hemothorax. 30.The response to catecholamines in an injured, hypovolemic pregnant woman can be expected to result in a. placental abruption.
b. fetal hypoxia and distress. c. fetal/maternal dysrhythmia. d. improved uterine blood flow. e. increased maternal renal blood flow. 31.A 5-year-old boy is struck by an automobile and brought to the emergency department. He is lethargic, but withdraws purposefully from painful stimuli. His blood pressure is 90 mm Hg systolic, heart rate is 140 beats per minute, and his respiratory rate is 36 breaths per minute. The preferred route of venous access in this patient is a. percutaneous femoral vein cannulation b. cutdown on the saphenous vein at the ankle. c. intraosseous catheter placement in the proximal tibia. d. percutaneous peripheral veins in the upper extremities. e. central venous access via the subclavian or interna1 jugular vein. 32.A 22-year-old man is brought to the hospital after crashing his motorcycle into a telephone pole. He is unconscious and in profound shock. He has no open wounds or obvious fractures. The cause of his shock is MOST LIKELY caused by a. a subdural hematoma. b. an epidural hematoma. c. a transected lumbar spinal cord. d. a transected cervical spinal cord. e. hemorrhage into the chest or abdomen. 33.A 42-year-old man is trapped from the waist down beneath his overtumed tractor for several hours before medical assistance arrives. He is awake and alert until just before arriving in the emergency department. He is now unconscious and responds only to painful stimuli by moaning. His pupils are 3 mm in diameter and symmetrically reactive to light. Prehospital personnel indicate that they have not seen the patient move either of his lower extremities. On examination in the emergency department, no movement of
his lower extremities is detected, even in response to painful stimuli. The most likely cause for this fmding is a. an epidural hematoma. b. a pelvic fracture. c. central cord syndrome. d. intracerebral hemorrhage. e. bilateral compartment syndrome. 34.All of the following signs on the chest x-ray of a blunt injury victim may suggest aortic rupture EXCEPT: a. mediastinal emphysema. b. presence of a "pleural cap." c. obliteration of the aortic knob. d. deviation of the trachea to the right. e. depression of the left mainstem bronchus 35.A young woman sustains a severe head injury as the result of a motor vehicular crash. In the emergency department, her GCS score is 6. Her blood pressure is 140/90 mm Hg and her heart rate is 80 beats per minute. She is intubated and is being mechanically ventilated. Her pupils are 3 mm in size and equally reactive to light. There is no other apparent injury. The most important principle to follow in the early management of her head injury is to a. administer an osmotic diuretic. b. prevent secondary brain injury. c. aggressively treat systemic hypertension. d. reduce metabolic requirements of the brain. e. distinguish between intracranial hematoma and cerebral edema. 36.A 25-year-old woman is brought to the emergency department after a motor vehicle crash. She was initially lucid at the scene and then developed a dilated pupil and contralateral extremity weakness. In the emergency department, she is unconscious and has a GCS score of 6. The initial management step for this patient should be to
a. obtain a CT scan of the head. b. administerdecadron 20 mg IV. c. perform endotracheal intubation. d. initiate an W line and administer Mannitol 1 g/kg. e. perform an emergency linar hole on the side of the dilated pupil. 37.Contraindication to nasogastric intubation is the presence of a a. gastric perforation. b. diaphragmatic rupture. c. open depressed skull fracture. d. fracture of the cervical spine. e. fracture of the cribriform plate. 38.A 24-year-old man sustains multiple fractured ribs bilaterally as a result of being crushed in a press at a plywood factory. Examination in the emergency department reveals a flail segment of the patient's thorax. Primary resuscitation includes high-flow oxygen administration via a nonrebreathing mask, and initiation of Ringer' s lactate solution. The patient exhibits progressive confusion, cyanosis, and tachypnea. Management at this time should consist of a. intravenous sedation. b. external stabilization of the chest wall. c. increasing the F102 in the inspired gas. d. intercostal nerve blocks for pain relief. e. endotracheal intubation and mechanical ventilation. 39.During resuscitation, which one of the following is the most reliable as a guide to volume replacement? a. Pulse rate b. Hematocrit c. Blood pressure d. Urinary output e. Jugular venous pressure
40.Which one of the following physical findings suggests a cause of hypotension other than spinal cord injury? a. priapism. b. bradycardia. c. diaphragmatic breathing. d. presence of deep tendon reflexes. e. ability to flex forearms but inability to extend them.
Advanced Trauma Life Support (ATLS) REVISED & VERIFIED EXAM STUDY GUIDE 2023 Urinary catheters are good for assessing renal perfusion and volume status. List 5 signs of urethral injury that might precent you from inser:ng one Blood at urethral meatus, perineal ecchymosis, blood in scrotum, high-riding/non-palpable prostate, pelvic fracture Which arm should you NOT put a pulse-ox on? The arm with the BP cuff 2 anatomical things that can interfere Obesity and intraluminal bowel gas Pa:ents with GSC of less than __8___ usually require intuba:on The "A" in ABCD stands for ______. Airway maintenance with Cervical SPINE protec:on You should assume that any pa:ent in a mul:system trauma with an altered level of consciousness or blunt injury above the clavice has what type of injury Cervical Spine Injury Flail chest is invariably accompanied by _______ which can interfere with blood oxygena:on pulmonary contusion - do NOT over fluid resuscitate these pa:ents. Hypotension is caused by ______ un:l proven otherwise. hypovolemia When you dont have a BP what are three things to look for when evalua:ng perfusion? 1. level of consciousness (brain perfusion 2. Skin color (ashen face/grey extremi:es) 3. Pulse (bilateral femoral - thready/tachy) Elderly pa:ents have a limited ability to ___________ to compensate for blood loss increase heart rate
Resuscita:on fluids should be warmed to 39 Celsius Only for Cyrstalloids, NOT for blood Urinary catheters are good for assessing renal perfusion and volume status. List 5 signs of urethral injury that might precent you from inser:ng one Blood at urethral meatus, perineal ecchymosis, blood in scrotum, high-riding/non-palpable prostate, pelvic fracture Which arm should you NOT put a pulse-ox on? The arm with the BP cuff 2 anatomical things that can interfere Obesity and intraluminal bowel gas When should radiographs be obtained? During the Secondary survey! How do get an ample pa:ent history? Allergies Medica:ons PMH/Pregnancy Last meal Events/Environment Why might you want a Bair Hugger for a pa:ent who smells of Alcohol? Vasodila:on can lead to hypothermia What things are you looking for when you do a DRE in trauma Blood, high riding prostate, sphincter tone, What should be done for every female pa:ent Pregnancy test Adult pa:ents should maintain UOP of at least _____mL/kg/hr. Kids should have at least ______ mL/kg/hr Adults 0.5 mL/kg.hr Kids 1.0 mL/kg/hr
Preven:ng hypercarbia (hypercapnia) is cri:cal in pa:ents who have sustained a _______ injury head What two places would you LOOK at a pa:ent if you suspect hypoxemia? Lips and fingernail beds Pa:ents may be abusive and belligerent because of ______-, so don't just assume its due to drugs, alcohol, or the fact they they are just inherently a jerk hypoxia Can a pa:ent breath on their own ager complete cervical cord transec:on Yes if the phrenic nerves (C3-C5 are spared. This will result in 'abdominal' breathing. The intercostal muscles will be paralyzed though Can you use an OPA (Guedel) in a conscious pa:ent No, it could make them vomit. An NPA (trumpet) would be okay. Bougies are typically inserted blindly, how do you know you are in the trachea and not the esophagus? You can feel the clicks as the distal :p rubs against the car:laginous tracheal rings, or it will deviate right or leg when entering either bronchus (at 50 cm) What do yo NOT want to hear if you auscultate a pa:ent ager placement of an ET tube? Borborygmi - rumbling or gurgling noices suggest esophageal inser:on. What is the RSI dose for etomidate 0.3 mg/kg (usually 20 mg) What is the RSI dose for succinylcholine 1-2 mg/kg (usually 100 mg) How does etomidate affect blood pressure it doesnt -it shouldnt have any effect on BP. Ketamine will increase BP, and propofol and thiopental will both drop BP.
A RSI dose of succinylcholine usually lasts about ___ minutes 5 What hypno:c/seda:ve/induc:on agent do you NOT want to use for a severely burned pa:ent? Sux - pa:ents with severe burns, crush injuries, hyperkalemia, or chronic paraly:c/neuromuscular disease should NOT get Sux because of hyperkalemia risk We have an expert-wriien solu:on to this problem! O2 should flow at 15L for needle cricothyroidotomy, and have a Y connector for insuffla:on if possible. What size needle do you use for adults? Kids? Adults: 12-14 gauge kids: 16-18 gauge Cricoid car:lage is the only circumferen:al support for the upper trachea in kids, therefore surgical cricothyroidotomy is not recommeded for kids under the age of ________. 12 In a 'normal' pa:ent without significant chest wall injury or lung disease, needle cricothyroidotomy can provide adequate oxygena:on for approximately ______ minutes 30-45 For a pa:ent with difficulty breathing, what things might you try before you provide a surgical aurway Chin Lig, jaw-thrust (NOT head-:lt while maintaining c-spine precau:ons), OPA (guedel), NPA (trumpet), LMA, Combitube, ET tube +- bougie How do you know if an OPA/Guedel is the correct size for the pa:ent? A correctly sized OPA will extend from the corner of the pa:ents mouth to the external auditory canal. What should you do with the balloon on an ET tube/LMA/foley before you insert it? Inflate it to make sure it doesnt leak- then deflate and insert We have an expert-wriien solu:on to this problem! What size LMA do you use for Kid, woman/small man, large woman/man Kid: 3 Woman/Small Man: 4
Large woman/Man 5 C3,4,5 keep them all alive The proper size ET tube for an infant is The same size as the infants nostril or liile finger (3 for neonates, 3.5 for infants What size cuffed endotracheal tube do you use for an emergency cricothyroidotomy? 5 or 6 Use a size 3 ET tube for neonates 3.5 for infants for 0-6 months 4 fo infants 6-12 months How do you calculate what size ET tube to use or toddlers and kids? Age/4 +4mm = internal diameter Shock is defined as an abnormality of the circulatory system that results in inadequate organ perfusion and :ssue oxygena:on. what are the 4 different types Neurogenic, cardiogenic, hypovolemic, sep:c The most common cause of shock in the injured trauma pa:ent is _________. hemorrhage Approximately ___% of the body's total blood volume is located in the venous circuit/ 70 Why does shock actually reduce the total volume of circula:ng blood? Anaerobic metabolism --> cant make more ATP --> endoplasmic then mitochondrial damage --> lysosomes rupture --> sodium and water enter the cell, which SWELLS and dies Which vasopressors should you use to treat hemorrhagic shock? What are the drug doses Never use pressors hypovolemic shock - use VOLUME replacement. Pressors will worsen :ssue perfusion in hemorrhagic shock Compensatory mechanisms may preclude a measurable fall in systolic blood pressure un:l up to ____% of the pa:ent's blood volume is lost. 30
Any pa:ent who is cool and is tachycardic is considered to be _______un:l proven otherwise. in shock The defini:on of tachycardia depends on pa:ents age. What heart rate is considered tachycardic for infants, toddlers/PS, schoolage/prepubescent, and adults Infants >160 toddlers/PS > 140 schoolage/prepubescent >120 Adults > 100 Elderly pa:ents may not exhibit tachycardia in response to hypovolemia because of limited cardiac response to catecholamines. Why else might they not get tachy? On a Beta-Blocker or have a pacemaker A FAST scan in an excellent way to diagnose cardiac tamponade. What signs sugget tamponade Beck's Triad: JVD, muffled heard sounds and hypotension (will be resistant to fluid therapy). Will also likely be tachycardic Pa:ents with a tension pneumo and pa:ent with cardiac tamponade may present with many of the same signs. What findings will you see with a tension will you NOT see with tamponade? Absent breath sounds and hyperresonance to percussion over the affected hemithorax. Immediate thoracic decompresion is warrented for anyone with absent breath sounds, hyperressonance to percussion, tracheal devia:on, _________ and _________ Acute respiratory distress & subcutaneous emphysema Can isolated intracranial injuries cause neurogenic shock? NO How do you calculate total blood volume in an adult? 70 mL per kg. A 70 kg person has about 5 L of circula:ng blood. (70*70) = 4900 mL How do you calculate TBV in child BW (kg) X 80-90 mL
The blood volume of an obese person is calculated based upon their _________ weight ideal Fluid replacement should be guided by __________, not simply by the ini:al classifica:on (Class I-IV) The pa:ent's response to ini:al replacement How much blood volume is lost with Class I hemorrhage? Up to 15% Dona:ng 1 pint, or ~500 mL of blood is about 10% volume lose and would classify as Class I Hemorrhage. Transcapillary refill and other compensatory mechanisms restore blood volume within 24 hours How much blood volume is lost with Class II hemorrhage> 15-30% (750-1500 mL in a 70 kg adult) How do you treat a Class II hemorrhage Usually just crystalloid resuscita:on Subtle CNS changes such as anxiety, fright, and hos:lity would be expected in pa:ent with a Class ____ hemorrhage II How much blood volume is lost with Class III hemorrhage? 30-40% (2000 mL in a 70 kg adult) A pa:ent with inadequate perfusion, marked tachycardia and tachypnea, significant mental status change, and a measurable fall in systolic blood pressure likely has a Class _____ Hemorrhage III or IV - these pa:ents requre a blood transfusion, which depends on their response to ini:al fluid resuscita:on. The first priority is stopping the hemorrhage. Loss of more than 50% of blood volume results in loss of consciousness. 50 How much blood volume is lost with Class IV hemorrhage? More than 40%. Unless very aggressive measures are taken the pa:ents will die within minutes
A Class _____ Hemorrhage represents the smallest volume of blood lost that is consistently associated with a drop in systolic blood pressure III Up to __________ mL of blood loss is commonly associated with femur fractures 1500 Unexplained hypotension or cardiac dsyrhythmias (usually bradycardia from excessive vagal s:mula:on) are ogen caused by ______ especially in children gastric disten:on How much crystalloid should you give an adult for an ini:al fluid resuscita:on bolus? for kids Adults: 2 L Kids: 20 mL/kg (may repeat and give as much as 60 mL/Kg but wit high reserve in kids, if they're in shock they should get blood sooner rather than later We have an expert-wriien solu:on to this problem! Each mL of blood loss would be replaced with ____ mL of crystalloid, thus allowing for replacement of plasma volume lot into inters::al and intracellular saces 3 Blood on the floor x four more is mneumonic for occult blood loss where? Chest, pelvis, retroperitoneum, and thigh For children UNDER 1 year of age, UOP should be ______ mL/Kg/Hr 2 Would pa:ents in EARLY hypovolemic shock be acidodic or alkalo:c? Alkalo:c - respiratory alkalosis from tachypnea .... followed later by mild metabolic acidosis in the early phase of shock "Rapid Responders" whose vital signs return to normal (and stay there) ager fluid I or II "Transient responders" are associated with Class ______ hemorrhage II or III
What differen:al diagnosis shoudl you always consider for "non-responders" following fluid resuscita:on? Non-hemorrhagic causes e.g. tension pneumothorax, tamponade, blunt cardiac injury, MI, acute gastric disten:on, neurogenic shock Most Pa:ents receiving blood transfusions ___________ need calcium replacement dont How should you posi:on the pa:ent before placing a subclavian or IJ line? Supine, head down 15 degrees to distend neck neck veins and prevent embolism, only turn head away is C-spine has been cleared first. How long can you keep and IO line in Intraosseous infusions should be limited to emergency resuscita:on and should be discon:nued as soon as other venous access is obtained Where do you want to make an incision for a saphenous vein cutdown and how long should your incisions be? 1 cm superior, 1 cm anterior to medial malleolus. 2.5 cm transverse incusion through the skin and SQ, careful to not to inure the vessel. A pa:ent arrives to the trauma bay intubated and there are absent breath sounds over the leg hemithorax, where should you place your decompression needle? This may NOT be a pneumothorax, for intubated pa:ents always suspect a right main-stem before aiemp:ng needle decompression. Where would you insert a large caliber needle to decompress a tension pneumo 2nd IC space in the midclavicular line of affected hemithorax For an open pneumothorax, (sucking chest wound) air passes preferen:ally through the chest wall defect (least resistance) if the diameter of the defect is at least ___ the diameter of the trachea. 2/3 Flail chest results from mul:ple rib fractures - by defini:on this would be ___ or more ribs, fractured in ___ or more places. 2 or more ribs fractured in 2 or more places
Both tension pneumothorax and massive hemothorax are associated with decreased breath sounds on ausculta:on, so you can tell which it is by _______. Percussion - hyperresonant with pnuemo, dull with hemothorax. If a pa:ent doesn't have JVD, does this mean they don't have a tension pneumo or tamponade? No, they might have a massive internal hemorrhage and be hypovolemic. By defini:on, how much blood is in the chest cavity to call it a "massive hemothorax"? 1500 mL or 1/3 or more of the pa:ent's total blood volume. (Some also define it as con:nued blood loss of 200 mL/hr for 2-4 hours- but ATLS does NOT use this rate for any mandatory treatment decisions). What size chest tube might you use to evacuate a massive hemothorax? #38 French - inserted at the 4th or 5th intercostal space, just anterior to the midaxillary line. What is Kussmaul's sign? A rise in venous pressure with inspira:on while breathing spontaneously, and is a true paradoxical venous pressure abnormality associated with cardiac tamponade. How well do CPR compressions work on someone with a penetra:ng chest injury and hypovolemia? "Closed heart massage for cardiac arrest or PEA is INEFFECTIVE in pa:ents with hypovolemia." Pa:ents with PENETRATING thoracic injuries who arrive pulseless, but with myocardial electrial ac:vity, may be candidates for an ED thoacotomy. Are pa:ents with PEA who have sustained blunt thoracic injuries candidates for an ED thoracotomy? NO - Only PEA with PENETRATING thoracic injuries should get an ED thoracotomy. An ED thoracotomy can allow you to do what? Evacuate pericardial blood, directly control hemorrhage, cardiac massage, cross-clamp the descending aorta to slow blood loss below the diaphragm and increase perfusion to the heart and brain. For a pa:ent with a trauma:c simple pneumothorax, what should you do BEFORE you start posi:ve pressure ven:la:on or take them to surgery for a GA?
Chest tube - posi:ve pressure ven:la:on can turn a sumple pneumo into a tension pneumo, so put in a chest tube first. Should you evacuate a simple hemothorax if it is not causing any respiratory problems? YES - A simple hemothorax, if not fully evacuated, may result in a retained, cloied hemothroax with lung entrapment or, if infected, develop into an empyema. We have an expert-wriien solu:on to this problem! A pneumothorax associated with a persistent large air leak ager tube thoracostomy suggests a _______ injury. tracheobronchial - Use bronchoscopy to confirm, you may need more than one chest tube before defini:ve opera:ve management. What radiographic findings are sugges:ve of trauma:c aor:c disrup:on? Widened medias:num, oblitera:on of aor:c knob, devia:on of trachea to the right, depression of leg mainstem bronchus, devia:on of esophagus (NG tube) to right, widened paratracheal stripe, fx'd 1st/2nd ribs or scapula. A decelera:on injury vic:m with a leg pnuemothorax or hemothorax without rib fractures, is in pain or shock out of propor:on to the apparent injury, and has par:culate maier in their chest tube may have _________. an ESOPHAGEAL RUPTURE - a forceful blow causes expulsion of gastric contents into the esophagus, producing a linear tear in the lower esophagus allowing leakage into the medias:num. Fractures for the lower ribs (10-12) should increase suspicion for _____ injury. hepatosplenic Why are upper torso, facial, and arm plethora with petechiae associated with crush injuries to the chest? Temporary compression of the superior vena cava. How does ATLS suggest you should review a chest radiograph? Trachea & bronchi, pleural spaces and parenchyma, medias:num, diaphragm, bones, sog :ssues, tubes & lines. You should use a size 16 or 18 gauge 6" needle for pericardiocentesis. How do you insert it? Puncture the skin 1-2 cm inferior to the leg xiphohondral junc:on at a 45 degree angle to the skin towards the heart, aiming toward the top of the leg scapula.
ATLS MCQ #1 A 22 year old man is hypotensive and tachycardic after a shotgun wound to the left shoulder. His blood pressure is initially 80/40 mmHg. After 2 liters of crystalloid solution his blood pressure increases to 122/84 mmHg His heart rate is now 100 beats per minute and his respiratory rate is 28 breaths per minute. His breath sounds are decreased in the left hemithorax, and after initial IV fluid resuscitation, a closed tube thoracostomy is performed for decreased left breath sounds with the return of a small amount of blood and no air leak. After chest tube insertion, the most approriate next step is a) reexamine the chest b) perform an aortogram c) obtain a CT scan of the chest d) Obtain arterial blood gas analyses e) perform tranesohageal echocardiography the answer is d ; but a website said a (and a makes more sense to me. so i'll go with a) info: chest tube insertion, p.108 ATLS MCQ #3 During the third trimester of pregnancy, all of the following changesoccur normally EXCEPT a a) decrease in PaCO2 b) decrease in leukocyte count c) reduced gastric emptying rate d) diminished residual lung volume e) diminished elvic ligament tension the answer is b. info: page 261 ATLS MCQ #4 In managing the head injured patient,the most important initial step is to a) secure the airway b. obtain c-spine film c) support circulation c) control scalp hemorrhage e) determine the GCS score the answer is a. info: p.154 ATLS MCQ #5 A previously healthy, 70kg (154 pound) man suffers an estimated acute blood loss of 2 liters. Which one of the following statements applies to this patient? a) his pulse pressure will be widened b) his urinary output will be at the lower limits of normal c) he will have tachycardia, but no change in his systolic blood pressure d) his systolic blood pressure will be decreased with a narrowed pulse ressure (true) e) his systolic blood pressure will be maintained with an elevated diastolicpressure the answer is d. info. page 61
ATLS MCQ #6 The physiologic hypervolemia of pregnancy has clinical significance in the management of the severely injured, gravid woman by a) reducing the need for blood transfusion b) increasing the risk of pulmonary edema c) complicating the management of closed head injury d) reducing the volume of crystalloid required for resuscitation e) increasing the volume of blood loss to produce maternal hypotension the answer is e. info. page 261 ATLS MCQ #7 The best guide for adequate fluid resuscitation of the burn patient is a) adequate urinary output b) reversal of systemic acidosis c) normalization of the heart rate d) a normal central venous pressure e) 4mL/kg/percent body burn/24 hours the answer is a. (not sure..) info: pg. 216-217 background: The adequacy of therapy is confirmed by simple determination of adequate urine output and of the haemoglobin and haematocrit levels. The most important guide is the patient’s clinical response source: http://www.medbc.com/annals/review/vol_16/num_4/text/vol16n4p173.asp ATLS MCQ #8 Establishing a diagnosis of shock must include a) hypoxemia b) acidosis c) hypotension d) increased vascular resistance e) evidence of inadequate organ perfusion the answer is e. the info: p.58 ATLS MCQ #9 A 7 year old boy is brought to the emergency department by his parents several minutes after he fell through a window. He is bleeding profusely from a 6-cm wound of his medial right thigh.Immediate management of the wound should consist of a) application of a tourniquet b) direct pressure on the wound c) packing the wound with gauze d) direct pressure on the femoral artery at the groin e) debridement of devitalized tissue the answer is: b info is: p.79; in obvious external bleeding section on table background
- tourniquet can be found on p.194, 195. they imply that you use a tourniquet if you are choosing life over limb, and if direct pressure isn't working. (p.195) ATLS MCQ #10 For the patient with severe traumatic brain injury, profound hypocarbia should be avoided to prevent a) respiratory alkalosis b) metabolic acidosis c) cerebral vasoconstriction with diminished perfusion d) neurogenic pulmonary edema e) shift of the oxyhemoglobin dissociation curve the answer is: c info is: related info on page 136,137 -------background info Carbon dioxide is perhaps the most potent available modulator of cerebrovascular tone and thus cerebral blood flow (CBF) http://www.liebertonline.com/doi/abs/10.1089/089771501750055776?journal Code=neu ----------Hypercarbia and hypoxia are both potent cerebral vasodilators that result in increased cerebral blood flow and volume and, potentially, increased ICP; thus, they must be avoided. Orotracheal intubation allows for airway protection in patients who are severely obtunded and allows for better control of oxygenation and ventilation. http://emedicine.medscape.com/article/909105-overview -----------ATLS MCQ #11 A 25 year old man is brought to a hospital with a general surgeon after being involved in a motor vehicle crash. He has a GCS of 13 and complains of abdominal pain. His blood pressure was 80mHg systolic by palpation on arrival at the hospital, but increases to 110/70 mmHg with the administration of 2 liters of intravenous fluid. His heart rate remains 120 beats per minute. Computed tomography shows an aortic injury and splenic laceration with free abdominal fluid. His blood pressure falls to 70mmHg after CT. The next step is a) contrast angiography b) transfer to higher level trauma center c) exploratory laparotomy d) transfuse packed red blood cells e) transesophageal echocardiography the answer is: c info: page 12 - list background info: none for now ATLS MCQ #12 Which one of the following statements regarding abdominal trauma in the pregnant patient is true? a) the fetus is in jeopardy only with major abdominal trauma b) leakage of amniotic fluid is an indication for hospital admission
c) indications for peritoneal lavage are different from those in the nonpregnant patient d) penetration of an abdominal hollow viscus is more common in late than in early pregnancy e) the secondary survey follows a different pattern from that of the nonpregnant patient the answer is: b info: page 265, i've marked it background info: none yet ATLS MCQ #13 The first maneuver to improve oxygenation after chest injury is a) intubate the patient b) assess arterial blood gases c) administer supplemental oxygen d) ascertain the need for a chest tube e) obtain a chest x-ray the answer is: c the info: can't find it! background: ATLS MCQ #14 A 25 year old man, injured in a motor vehicular crash, is admitted to the emergency department. His pupils react sluggishly and his eyes open to painful stimuli. He does not follow commands, but he does moan periodically. His right arm is deformed and does not respond to painful stimulus; however, his left hand reaches purposefully toward the painful stimulus. Both legs are stiffly extended. His GCS Score is a) 2 b) 4 c) 6 d) 9 e) 12 the answer is: d info: p.138 background info: i used my card on gcs for this. ATLS MCQ #15 A 20 year old woman, at 32 weeks gestation, is stabbed in the upper right chest. In the emergency department,her blood pressure is 80/60 mmHg. She is gasping for breath, extremely anxious, and yelling for help. Breath sounds are diminished in the right chest. The most appropriate first step is to a) perform tracheal intubation b) insert an oropharyngeal airway c) perform needle decompression of the right chest d) manually displace the gravid uterus to the left side of the abdomen e) initiate 2, large-caliber peripheral IV lines and crystalloid infusion the answer is: c info is: p.87 background is: ATLS MCQ #16
Which one of the following findings in an adult should prompt immediate management during the primary survey? a) distended abdomen b) glasgow coma scale score of 11 c) temperature of 36.5C (97.8F) d) heart rate of 120 beats per minute e) respiratory rate of 40 breaths per minute the answer is: e the info is: page 79; topic on shock; addresses distended abdomen, not RR
ATLS MCQ #17 The most important, immediate step in the management of an open pneumothorax is a) endotracheal intubation b) operation to close the wound c) placing a chest tube through the chest wound d) placement of an occlusive dressing over the wound e) initiation of 2, large-caliber IVs with crystalloid solution the answer is; d the info is: page 87 background is: ATLS MCQ #18 The following are contraindications for tetanus toxoid administration a) history of neurological reaction or severe hypersensitivity to the product b) local side effects c) muscular spasms d) pregnancy e) all of the above the answer is: a the info is: some related stuff on page 297 background info: It is a contraindication to use this or any other related vaccine after a serious adverse event temporally associated with a previous dose including an anaphylactic reaction.A history of systemic allergic or neurologic reactions following a previous dose of Tetanus Toxoid is an absolute contraindication for further use.2,5 found a source http://www.rxlist.com/tetanus-drug.htm ATLS MCQ #19 A 56 year old man is thrown violently against the steering wheel of his truck during a motor vehicle crash. On arrival in the emergency department he is diaphoretic and complaining of chest pain. His blood pressure is 60/40 mmHg and his respiratory rate is 40 breaths per minute. Which of the following best differentiates cardiac tamponade from tension pneumothorax as the cause of his hypotension? a. tachycardia b. pulse volume c. breath sounds d. pulse pressure
e. jugular venous pressure the answer is: c info is: p.87 marked background info:
ATLS MCQ #20 Bronchial intubation of the right or left mainstem bronchus can easily occur during infant endotracheal intubation because a) the trachea is relatively short b) the distance from the lips to the larynx is relatively short c) the use of tubes without cuffs allows the tube to slip distally d) the mainstem bronchi are less angulated in their relation to the trachea e) so little friction exists between the endotracheal tube and the wall of thetrachea the answer is: a the info: p228 marked background info: ATLS MCQ #21 A 23 year old man sustains 4 stab wounds to the upper right chest during an altercation and is brought by ambulance to a hospital that has full surgical capabilities. His wounds are all above the nipple. He is endotracheally intubated, closed tube thoracostomy is performed, and 2 liters of crystalloid solution are infused through 2 large caliber IVs. His blood pressure now is 60/0 mmHg, heart rate is 160 beats per minute, and respiratory rate is 14 breaths per minute (ventilated with 100% O2). 1500cc of blood has drained from the right chest. The most appropriate next step in managing this patient is to a) perform FAST b) obtain a CT of the chest c) perform an angiography d) urgently transfer the patient to the operating room e) immediately transfer the patient to a trauma center the answer is: d the info is: read p.90-91 background info: ATLS MCQ #22 A 39 year old man is admitted to the emergency department after an automobile collision. He is cyanotic, has insufficient respiratory effort, and has aGCS score of 6. His full beard makes it difficult to fit the oxygen facemask to his face. the most appropriate next step is to a) perform a surgical cricothyroidotomy b) attempt nasotracheal intubation c) ventilate him with a bag-mask device until c-spine injury can be excluded d) attempt orotracheal intubation using 2 people and inline stabilization of the cervical spine. e) ventilate the patient with a bag-mask device until his beard can be shaved for better mask fit. the answer is: d
the info is: related to anaswer p.33; marked background info is: ATLS MCQ #23 A patient is brought to the emergency department 20 minutes after a motor vehicle crash. He is conscious and there is no obvious external trauma. He arrives at the hospital completely immobilized on a long spine board. His blood pressure is 60/40 mmHg and his heart rate is 70 beats per minute. His skin is warm. Which one of the following statements is true? a) vasoactive medications have no role in the patient's management b) the hypotension should be managed with volume resuscitation alone c) flexion and extension views of the c spine should be performed early d) occult abdominal visceral injuries can be excluded as a cause of hypotension e) flaccidity of the lower extremities and loss of deep tendon reflexes are expected the answer is: c the info is: related topics on p.167; 161 background: ATLS MCQ #24 Which one of the following is the recommended method for initialy treating frostbite? a) moist heat b) early amputation c) padding and elevation d) vasodilators and heparin e) topical application of silver sulfadiazine the answer is: a the info is: p.220 background is: "4. Use moist heat (warm water) to warm the casualty, rather than dry heat (radiator or fire). Dry heat can cause more damage." source http://ezinearticles.com/?10Tips-For-Treating Frostbite&id=1666183 ATLS MCQ #25 A 32 year old man's right leg is trapped beneath his overturned car for nearly 2 hours before he is extricated. On arrival in the emergency departmnet, his right lower extremity is cool, mottled, insensate, and motionless. Despite normal vital signs, pulses cannot be palpated below the femoral vessel and the muscles of the lower extremity are firm and hard. During the initial management of this patient, which of the following is most likely to improve the chances for limb salvage? a) applying skeletal traction b) administering anticoagulant drugs c) administering thrombolytic therapy d) perform right lower extremity fasciotomy e) immediately transferring the patient to a trauma canter the answer is: d info is: related on p196-197 background is:
ATLS MCQ #26 A patient arrives in the emergency department after being beaten about the head and face with a wooden club. He is comatose and has a palpable depressed skull fracture. His face is swollen and ecchymotic. He has gurgling respirations and vomitus on his face and clothing. The most appropriate step after providing supplemental oxygen and elevating his jaw is to a) requires a CT scan b) insert a gastric tube c) suction the oropharynx d) obtain a lateral cervical spine x-ray e) ventilate the patient with a bag-mask the answer is: c the info is: i can't find it ATLS MCQ #27 A 22 year old man sustains a gunshot wound to the left chest and is transported to a small community hospital at which surgical capabilities are not available. In the emergency department, a chest tube is inserted and 700mL of blood is evacuated. The trauma center accepts the patient in transfer. Just before the patient is placed in the ambulance for transfer, his blood pressure decreases to 80/68 mmHg and his heart rate increases to 136 beats per minute . The next step should be to a) clamp the chest tube b) cancel the patient's transfer c) perform an emergency department thoracotomy d) repeat the primary survey and proceed with transfer e) delay the transfer until the referring doctor can contact a thoracic surgeon the answer is:c the info is: some reference can be found btwn page 270 - 274 ATLS MCQ #28 A 64 year old man, involved in a high speed car crash, is resuscitated initially in a small hospital with limited resources. He has a closed head injury with a GCS score of 13. He has a widened mediastinum on chest x-ray with fractures of left ribs 2 through 4, but no pneumothorax. After infusing 2 liters of crystalloid solution, his blood pressure is 100/74 mmHg, heart rate is 110 beats per minute, and respiratory rate is 18 breaths per minute. He has gross hematuria and a pelvic fracture. You decide to transfer this patient to a facility capable of providing a higher level of care. The facility is 128 km (80 miles) away. Before transfer, you should first a) intubate the patient b) perform diagnostic peritoneal lavage c) apply the pneumatic antishockgarment d) call the receiving hospital and speak to the surgeon on call e) discuss the advisability of transfer with the patient's family the answer is: c the info is: related on p.123 background: ATLS MCQ #29 Hemorrhage of 20% of the patient's blood volume is associated usually with
a) oliguria b) confusion c) hypotension d) tachycardia e) blood transfusion requirement the answer is; d info is: page 61 background: ATLS MCQ #30 Which one of the following statements concerning intraosseous infusion is true? a) only crystalloid solutions may be safely infused through the needle (NO) b) aspiration of bone marrow confirms appropriate positioning of the needle (POSSIBLE) c) intraosseous infusion is the preferred route for volume resuscitation in small children (NO) d) intraosseous infusion may be utilized indefinitely (NO) e) swelling in the soft tissues around the intraosseous site is not a reason to discontinue infusion the answer is: b the info: random unhelpful info p.236 background: "This route of fluid and medication administration is an alternate one to the preferred intravascular route when the latter can't be established in a timely manner especially during pediatric emergencies. When intravascular access cannot be obtained in pediatric emergencies, intraosseous access is usually the next approach." ". It can be maintained for 24–48 hours, after which another route of access should be obtained." the info is: "Furthermore, any medication that can be introduced via IV can be introduced via IO." source: http://en.wikipedia.org/wiki/Intraosseous_infusion
ATLS MCQ #31 A young woman sustains a severe head injury as the result of a motor vehicle crash. In the emergency department, her GCS is 6. Her blood pressure is 140/90 mmHg and her heart rate is 80 beats per minute. She is intubated and mechanically ventilated. Her pupils are 3mm in size and equally reactive to light. There is no other apparent injury. The most important principle to follow in the early management of her head injury is to a) avoid hypotension b) administer an osmotic diuretic c) aggressively treat systemic hypertension d) reduce metabolic requirements of the brain e) distinguish between intracranial hematoma and cerebral edema the answer is: A the info is: page 142,143, #145 background: none for now
ATLS MCQ #32 A 33 year old woman is involved in a head-on motor vehicle crash. It took 30 minutes to extricate her from the car. Upon arrival in the emergency department, her heart rate is 120 beats per minute, BP is 90/70 mmHg, respiratory rate is 16 breaths per minute, and her GCS score is 15. Examination reveals bilaterally equal breath sounds, anterior chest wall ecchymosis, and distended neck veins. Her abdomen is flat, soft, and not tender. Her pelvis is stable. Palpable distal pulses are found in all 4 extremities. Of the following, the most likely diagnosis is a) hemorrhagic shock b) cardiac tamponade c) massive hemothorax d) tension pneumothorax e) diaphragmatic rupture the answer is: b the info is: p.91-92 background is: ATLS MCQ #33 A hemodynamically normal 10 year old girl is admitted to the Pediatric Intensive Care Unit (PICU) for observation after a Grade III(moderately severe) splenic injury has been confirmed by computed tomography (CT). Which of the following mandates prompt celiotomy (laparotomy)? a) A serum amylase of 200 b) A leukocyte count of 14,000 c) extraperitoneal bladder rupture d) free intraperitoneal air demonstrated on follow-up CT e) a fall in the hemoglobin level from 12 g/dL to 8 g/dL over 24 hours the answer is: e the info is: some related info p.121 under solid organ injuries background: ATLS MCQ #34 A 40 year old woman restrained driver is transported to the emergency department in full spinal immobilization. She is hemodynamically normal and found to be paraplegic at the level of T10. Neurologic examination also determines that there is loss of pain and temperature sensation with preservation of proprioception and vibration. These findings are consistent with the diagnosis of a) central cord syndrome b) spinal shock syndrome c) anterior cord syndrome d) complete cord syndrome e) Brown-Sequard syndrome the answer is: c the info is: p.163 background: ATLS MCQ #35 A trauma patient presents to your emergency department with inspiratory stridor and a
suspected c-spine injury. Oxygen saturation is 88% on high-flow oxygen via a nonrebreathing mask. The most appropriate next step is to: a) apply cervical traction b) perform immediate tracheostomy c) insert bilateral thoracostomy tubes d) maintain 100% oxygen and obtain immediate c-spine x-rays e) maintain inline immobilization and establish a definitive airway the answer is: e the info is: related info p.27-28 background: some info on flashcards for tracheostomy indications (i.e. you don't do it in emergency cases) ATLS MCQ #36 When applying the Rule of Nines toinfants, a) It is not reliable b) the body is proportionally larger in infants than in adults c) the head is proportionally larger in infants than in adults d) the legs are proportionally larger in infants than in adults e) the arms are proportionally larger in infants than in adults the answer is: c the info is: p.223 background info: ATLS MCQ #37 A 60 year old man sustains a stab wound to the right posterior flank. Witnesses state the weapon was a small knife. His heart rate is 90 beats per minute, blood pressure is 128/72 mmHg, and respiratory rate is 24 breaths per minute. The most appropriate action to take at this time isto a) perform a colonoscopy b) perform a barium enema c) perform an intravenous pyelogram d) perform serial physical examination e) suture repair the wound and outpatient follow up the answer is: d the info is: i can't find it background: slightly helpful is flashcards on kidney injury ATLS MCQ #38 Which of the following situations requires Rh immunoglobulin administration to an injured woman? a) Negative pregnancy test, Rh negative, and torso trauma b) positive pregnancy test, Rh positive, and has torso trauma c) positive pregnancy test, Rh negative, and has torso trauma d) positive pregnancy test, Rh positive, and has an isolated wrist fracture e) positive pregnancy test, Rh negative, and has an isolated wrist fracture the answer is: c
the info is: p.265 background is: ATLS MCQ #39 A 22 year old female athlete is stabbed in her left chest at the third interspace in the anterior axillary line. On admission to the emergency department and 15 minutes after the incident, she is awake and alert. Her heart rate is 100 beats per minute, blood pressure 80/60 mmHg, and respiratory rate 20 breaths per minute. A chest x-ray reveals a large left hemothorax. A left chest tube is placed with an immediate return of 1600 mL of blood. the next management step for this patient is a) perform a thoracoscopy b) perform an arch aortogram c) insert a second left chest tube d) prepare for an exploratory thoracotomy e) perform an chest CT the answer is: d the info is: p.90-91 background: ATLS MCQ #40 A 6 year old boy walking across the street is struck by the front bumper of a sports utility vehicle traveling at 32 kph (20 mph). Which one of the following statements is true? a) a flail chest is probable b) a symptomatic cardiac contusion is expected c) a pulmonary contusion may be present in the absence of rib fractures d) transection of the thoracic aorta is more likely than in an adult patient e) rib fractures are commonly found in children with this mechanism of injury the answer is: c the info is: p.237 background:
1. Which following LEAST diagnostic esophageal
therapy should be administered to
intubation ? ETT above carina on chest x-
pregnant females who have sustained a gunshot wound to the leg 9. 18-year-old motorcyclist, massive facial injuries
trauma patient? severe maxillofacial
in a head crash with pick up, 150/88, ** beats,
vi a
ray 2. Which following signs definitive airway in fractures 3. Twenty-seven patient injured aircraft local
RR 26 labored and sonor, GCS 7, orotracheal
d
intubation unsuccesfull, be apatic, best
airport. Principal triage? produce the
sh ar e
procedure? surgical cricothyroidotomy 10.25-year-old partial and full thickness burn
greates number of survivor based on available resources 4. Statement correct? Cerebral contusions
involving 60% right arm and hand, circumferential burn, pulse absent right wrist
may coalesce to form an intracerebral
as
and not detected by Doppler examination, first
and right posterior axillary line, TD 11060, 90
s ur tu se dy H er res o. ou co rc m e
beat, RR 34, airways and double IV line, next
appropriate step? obtain a portable chest
x-ray 6. A 47-year-old, falling 6 meters (20 feet) landing straddle on a face, perineum reveals extensive echymosis, blood MAE, initial diagnostic study for evaluation urinary track ? retrograde
urethrography 7. Neurogenic shock following classic characteristic except which ? narrowed pulse pressure 8. Following false concerning Rh isoimmunization
Th
is
in pregnant trauma ? Rh immunoglobulin
This study source was downloaded by 100000812856574 from CourseHero.com on 11-10-2021 14:37:24 GMT -06:00
step manage right upper? escharotomy 11. Following sign chst x-ray patient a blunt
w
hematoma 5. An 18-yearold, shot, bullet wound right clavicula
injury suggest aortic rupture EXCEPT which ?
mediastinal emphysema 12.30-year-old woman fell down four stairs landing on concrete. Uncon scious for 5 ,imutes after the fall, full consciousness during 10 minute transport the hospital, GCS 15, complaint is a slight headache, 30 minute she unresponsive GCS 6 left pupil is large ? an epidural hematoma 13.24-year-old male pedestrian, struck automobile, admit EmerDeptt 1 hour after injury, 80/60, 140 beat, RR 36, lethargic, oxygen face mask, 2 IV
line, BGA PaO2 118, PaCO2 30, PH 7,21. Best
reveal persistant right pneumothorax, most
accomplished by ? restoration of normal
likely cause persistent right pneumothorax ? tracheobronchial injury 19.22 year-old female 6 month pregnant motor
vi a
perfusion 14. Which following first any patient may include multiple close extremity fracture?
crash, vaginal bleeding. Initial step treatment ? ask the patient what her name is 20.Construction worker falls a scaffold, HR 124
ensuring adequate oxygenation and
d
ventilation. 15. A 30-year-old gubshot wound right lower
sh ar e
beat, 85/60, lower abdominal pain airway, chest, immobilizing spine, initial fluid, next step?
hest, midway nipple and costal margin, in EmerDeptt he endotracheally intubated, 2 liters
FAST exam 21.22-year-old male a shotgun wound left shoulder
chrystaloid two IV line, closed tube
and chest at close range, 80/40, 130 beat, after
as
thoracostomy 200 ml of blood, his blood now
2 liters fluid 122/84, 100 beat, tachypneu 28
a laparotomy in the operating room 16. You are treating trauma patient definitive
breath sound decrease upper chest with
w
70/0, 140 beats, most appropriate? perform
dullness on percussion, large caliber tube thoracostomy insert return 200 ml blood, most
valuable for succesfull intubation ? gum
appropriate next step? repeat the physical
s ur tu se dy H er res o. ou co rc m e
airway by intobation, vocal cord not vsible. Most elastic bougle 17. 79-year-old female motor crash, she on
examination of the chest 22. Which following concerning spine and
Coumadin and a beta blocker. Following true ?
cord is TRUE ? diaphragmatic breathing in
vigorous fluid resuscitation may be
an unconscious patient who has fallen is
associated with cardiorespiratory failure 18.22-year-old male falling from 2,4 meter (8 feet),
sign of spine injury 23. Which following TRUE ? nasotracheal
large right pneumothoraxchest tube inserted
tube positions a cuffed airway in the
connected drainage, chest x-ray demonstrated
infraglottic space
Th
is
residual large right pneumothorax, third x-ray
This study source was downloaded by 100000812856574 from CourseHero.com on 11-10-2021 14:37:24 GMT -06:00
32.Compared with adults, children have ? lower
airway clear, RR 28, systole 140pain on
incidence of bony injury with neurogenic
palpation of chest ? pain management 25. Most common acid-base disturbance
shock 33.30 year-old male motor crash, RR 18, 88 beats,
vi a
24.40-year-old male fall from 3 meter (10 feet),
encountered in injured pediatric patient ?
130/72, GCS 13. Laparotomy indicates when ? CT demonstrated retroperitoneal air 34.20-year-old male 30 minute after stabbed in the
d
changes in ventilation 26. 17-year-old femae 2 meters fall,
chest, 3 cm wound medial left nipple. 70/33,
treatment? applying oxygen and
140 beat, neck and arm vein distended, breath
sh ar e
unresponsive and RR 32, 90/60, 68 beat. First maintaining airway 27. Which one following TRUE regarding
sound normal, heart sound diminished, IV access warm crystalloid. Manage? FAST
diagnostic peritoneal lavage? DPL has a
as
exam 35.35-year-old motorcyclist a frontal impact
hemoraahagic shock ( 750-1500 ml ) heart
s ur tu se dy H er res o. ou co rc m e
rate above 140 beats per minutes 29. Neurogenic shock is ? initially managed with vasopressor therapy 30.23 year-old male full-thickness burns his
head,arms, upper torso, total 50%, weight 80 kg 105/75, 135 beat, cateter urine 20 ml dark,
received 1000 ml RLParkland guide estimate
crystalloid fluid next 8 hours ? 1000 ml 31.34 –year-old female motor crash, talking, voice is hoarse, bruishing chest and anterior neck, next step? oxygen by non-rebreathing
Th
is
mask
This study source was downloaded by 100000812856574 from CourseHero.com on 11-10-2021 14:37:24 GMT -06:00
collision, 140 beat, 86/60, RR 36, bitterly lower
w
high sensitivity 28. Which following sign class II
abdominal pain, a leg-length discrepancy and external rotations left leg. Which following TRUE ? x-ray of the chest and pelvic are
important in the initial evaluation 36.35-year-old female fall down stairs, extensive bruising face and head, 120 beat, 90/70, RR 26, condition most readily? hypovolemia from abdominal or pelvic injury 37. Which following TRUE cranial anatomy? the choroid plexus, which produced cerebrospinal fluid, lies in the lateral and third ventricles.
38.22-year-old woman fall skiing, present spine
eyes, verbalizing inappropriate words, pupil
board cervical collar, oxygen mask 5 Lpm,
equal, most important step? immediate
double IV, GCS 12, 135/76, 105 beat, RR 19.
intubation to protect his airways 40.In a patient with a spinal cord injury, sacral
vi a
Manage ? CT of the head and repeat GCS 39.A young male fall 5 meters (16 feet) from roof,
sparing? is a good prognostic sign.
Th
is
s ur tu se dy H er res o. ou co rc m e
w
as
sh ar e
d
respond pain by pushing your hand, opening
This study source was downloaded by 100000812856574 from CourseHero.com on 11-10-2021 14:37:24 GMT -06:00 Powered by TCPDF (www.tcpdf.org)